0% found this document useful (0 votes)
23 views100 pages

SP7 Past Papers 2

The document outlines the examination details for Subject ST7 – General Insurance: Reserving and Capital Modelling, conducted by the Institute and Faculty of Actuaries on April 16, 2012. It includes instructions for candidates, a series of questions covering various topics related to general insurance, and an examiner's report analyzing candidate performance and areas of improvement. Key areas of focus include copula functions, fraudulent claims, investment policy constraints, balance sheet construction, and the use of capital models in planning processes.

Uploaded by

Dani Sha
Copyright
© © All Rights Reserved
We take content rights seriously. If you suspect this is your content, claim it here.
Available Formats
Download as PDF, TXT or read online on Scribd
0% found this document useful (0 votes)
23 views100 pages

SP7 Past Papers 2

The document outlines the examination details for Subject ST7 – General Insurance: Reserving and Capital Modelling, conducted by the Institute and Faculty of Actuaries on April 16, 2012. It includes instructions for candidates, a series of questions covering various topics related to general insurance, and an examiner's report analyzing candidate performance and areas of improvement. Key areas of focus include copula functions, fraudulent claims, investment policy constraints, balance sheet construction, and the use of capital models in planning processes.

Uploaded by

Dani Sha
Copyright
© © All Rights Reserved
We take content rights seriously. If you suspect this is your content, claim it here.
Available Formats
Download as PDF, TXT or read online on Scribd
You are on page 1/ 100

INSTITUTE AND FACULTY OF ACTUARIES

EXAMINATION

16 April 2012 (pm)

Subject ST7 – General Insurance:


Reserving and Capital Modelling
Specialist Technical

Time allowed: Three hours

INSTRUCTIONS TO THE CANDIDATE

1. Enter all the candidate and examination details as requested on the front of your answer
booklet.

2. You have 15 minutes at the start of the examination in which to read the questions.
You are strongly encouraged to use this time for reading only, but notes may be made.
You then have three hours to complete the paper.

3. You must not start writing your answers in the booklet until instructed to do so by the
supervisor.

4. Mark allocations are shown in brackets.

5. Attempt all seven questions, beginning your answer to each question on a separate
sheet.

6. Candidates should show calculations where this is appropriate.

AT THE END OF THE EXAMINATION

Hand in BOTH your answer booklet, with any additional sheets firmly attached, and this
question paper.

In addition to this paper you should have available the 2002 edition of the Formulae
and Tables and your own electronic calculator from the approved list.

ST7 A2012 © Institute and Faculty of Actuaries


1 (i) Define a copula function and suggest the rationale for its actuarial use in
general insurance. [2]

(ii) Derive copula expressions for variables X and Y when these are not
statistically independent. [3]

(iii) State the copula expression used most commonly in insurance applications and
summarise its characteristics. [2]
[Total 7]

2 (i) Outline quantitative indicators of an increase in fraudulent claims in a motor


insurance portfolio. [3]

(ii) Describe how an insurer could attempt to reduce the number of fraudulent
claims. [5]
[Total 8]

3 Describe the main constraints a general insurer faces in setting its investment policy.
[10]

4 Two general insurance companies A and B operate in the same country. Each
company writes primarily personal lines motor and property business. The following
information relates to the year ending 31 December 2011.
A B

Gross Written Premium 152 1,112


Additional Unexpired Risk Reserve c/fwd 47 0
Gross Outstanding Claims Reserve b/fwd 61 3,203
Gross Claims Paid 111 647
Gross Outstanding Claims Reserve c/fwd 94 3,076
Non Acquisition Expenses 17 129
Investment Income 10 68
Current Assets at 31/12/2011 17 133
Current Liabilities at 31/12/2011 35 171
Investments at 31/12/2011 386 4,372
Share Capital at 31/12/2011 46 541
Acquisition Costs as a % of Gross Written Premium 38% 23%

(i) Construct the balance sheet for each of the two companies as at 31 December
2011, stating any assumptions made. [4]

(ii) Derive underwriting, solvency and return on capital employed ratios for each
company, ignoring taxation, stating any assumptions made. [5]

(iii) Comment on the results in part (ii). [4]


[Total 13]

ST7 A2012–2
5 The following claims information for a large insurer has been provided:

Under- Outstanding Selected Selected


Paid Loss
writing Premium Case Incurred Ultimate
Claims Ratio
Year Reserves Development Cost

2005 109,900 2,700 19,700 100.0% 22,400 20.4%


2006 103,000 6,300 19,300 102.3% 25,027 24.3%
2007 78,300 9,000 12,100 109.4% 19,285 24.6%
2008 78,700 10,100 9,300 104.6% 18,548 23.6%
2009 80,200 13,300 5,400 84.2% 22,207 27.7%
2010 82,100 8,000 1,000 21.9%

Premium Rate Increases

2005 −6%
2006 −8%
2007 −6%
2008 0%
2009 6%
2010 5%

Claims Inflation
Assumption 5%

(i) Discuss the advantages and disadvantages of the following approaches for
deriving an estimated loss ratio for use within the Incurred Bornhuetter-
Ferguson method:

(a) Using the underwriter’s view of the priced loss ratio.

(b) Using market loss ratios derived from industry benchmark information.

(c) Using an average of the last three years selected ultimate loss ratios
adjusted for premium rate increases and claims inflation.
[5]

(ii) Using method (c) above, estimate the ultimate loss ratio for underwriting year
2010. [3]

(iii) Comment on the ultimate loss ratio estimates by underwriting year and any
alternative projection methods that may be appropriate. [2]
[Total 10]

ST7 A2012–3 PLEASE TURN OVER


6 A small but expanding general insurance company has over the last three years
written a packaged product for small businesses. The product includes property,
employers’ and public liability and business interruption covers. The basis for the
outstanding claims reserve for the product is being reviewed.

(i) Compare the relative merits of case estimates and statistical methods when
calculating a reserve for outstanding claims for this product. [8]

(ii) For each of the property, liability and business interruption covers, outline the
characteristics of the claims expected under this product [7]

When grouping the data for the purpose of statistical reserving for outstanding claims,
the company currently combines all property and business interruption claims but
treats liability claims separately.

(iii) Discuss the advantages and disadvantages of this approach. [6]

(iv) Explain the circumstances in which an exposure-based method would be


appropriate for estimating the reserve and how it would be applied. [3]

(v) Describe how the business environment might impact the basis and method for
estimating the reserve. [3]
[Total 27]

7 An insurer writes only public and employers’ liability business. The company actuary
has been asked by the managing director to consider ways to improve the internal
planning process by including output from the capital model.

(i) Discuss how the actuary could use the capital model to inform the planning
process and which parts of the business may benefit from this. [7]

The managing director is particularly interested in including information from the


insurance risk element of the model as the company is planning to start writing
commercial motor business.

(ii) Discuss how you could parameterise the insurance risk elements of this capital
model for this new class of business, considering any practical issues faced.
[12]

(iii) Discuss the potential impact on the company of the use of inappropriate model
parameters. [6]
[Total 25]

END OF PAPER

ST7 A2012–4
INSTITUTE AND FACULTY OF ACTUARIES

EXAMINER’S REPORT

April 2012 examinations

Subject ST7 – General Insurance:


Reserving and Capital Modelling
Specialist Technical

Purpose of Examiners’ Reports

The Examiners’ Report is written by the Principal Examiner with the aim of helping
candidates, both those who are sitting the examination for the first time and who are using
past papers as a revision aid, and also those who have previously failed the subject. The
Examiners are charged by Council with examining the published syllabus. Although
Examiners have access to the Core Reading, which is designed to interpret the syllabus, the
Examiners are not required to examine the content of Core Reading. Notwithstanding that,
the questions set, and the following comments, will generally be based on Core Reading.

For numerical questions the Examiners’ preferred approach to the solution is reproduced in
this report. Other valid approaches are always given appropriate credit; where there is a
commonly used alternative approach, this is also noted in the report. For essay-style
questions, and particularly the open-ended questions in the later subjects, this report contains
all the points for which the Examiners awarded marks. This is much more than a model
solution – it would be impossible to write down all the points in the report in the time allowed
for the question.

T J Birse
Chairman of the Board of Examiners

July 2012

© Institute and Faculty of Actuaries


Subject ST7 (General Insurance: Reserving and Practical Modelling Specialist Technical)
– Examiners’ Report – April 2012

General comments on Subject ST7

Candidates who are well prepared generally appear to perform reasonably on ST7, with the
more challenging questions tending to occur on SA3. Candidates should consider the
following advice however (if they are not already):

• Lists are hugely valuable for breadth of point generation but candidates should always
exercise judgement when applying them.

• Calculation questions will come up on a regular basis with ST7, as candidates can clearly
observe from examination of historical papers. Candidates should always be prepared for
such staples as balance sheet preparation, triangle manipulations & projections and
reinsurance layer calculations (along with being able to carry out any necessary
adjustments including inflation, exposure and time period issues).

• Capital questions should be expected on every paper and represent a sufficient proportion
of the course content that candidates should not expect to be able to pass on their
reserving knowledge alone. Those who do not encounter capital work in their
professional lives should be particularly careful to ensure that they take time to
familiarise themselves with this element of the course.

• Candidates should aim to be able to give near exact glossary definitions as incoherent or
vague descriptions will be marked harshly. If candidates struggle to remember definitions
verbatim they should take the time to properly analyse the glossary definition to ensure
they have fully absorbed all the nuances of the definition.

• It is important to always read the question properly.

Comments on the April 2012 paper

This was a fairly standard paper and none of the questions should have been unexpected for
well prepared candidates. Performance however was disappointing, reflected in the lowest
pass rate observed since April 2006.

Key areas of poor performance were:

Q1 – straight bookwork that most candidates clearly had not learnt.

Q4 – candidates should be entirely prepared for balance sheet questions yet many made
obvious and avoidable mistakes.

Q5 – candidates did not take the time to question what the underlying business might be or
comment accordingly. Many missed the purpose of the loss ratio required.

Q6 – few candidates made any effort to tie their answers to the specifics of the question.

Page 2
Subject ST7 (General Insurance: Reserving and Practical Modelling Specialist Technical) –
April 2012 – Examiners’ Report

Q7 – as in previous papers, few candidates seemed to understand capital or even to have


learnt the bookwork. Candidates should note that capital modelling is a key part of ST7 and
unless they perform extremely strongly on reserving questions they will struggle to pass ST7
without some understanding of capital modelling.

Page 3
Subject ST7 (General Insurance: Reserving and Practical Modelling Specialist Technical)
– Examiners’ Report – April 2012

1 (i) A copula is a mathematical relationship between the individual distributions of


random variables
… and the joint distribution of those variables.
Copulas allow greater flexibility when multiple dependencies are modelled
than single correlation factors allow or, alternatively, enable a greater level of
dependency in the tail
E.g. a cat event or similar affects the tail of the distribution and will impact a
number of classes and hence give correlation in the tail

(ii) Given two variables X and Y.

Let FX(x) = P(X≤x) and FY(y) = P(Y≤y) be the respective cumulative


distribution functions.

X and Y are not independent so the joint distribution function needs to be


considered

FXY(x,y) = P(X ≤ x, Y ≤ y)

A copula function (C) is a function that allows calculation of the joint


distribution function from the values of the marginal distributions so that

FXY(x,y) = C[FX(x), FY(y)]

Define C(u,v) = F(F−1(u), F−1(v))

Hence C(FX(x), FY(y)) = F(F−1(FX(x)), F−1(FY(y))) = F(x,y)

The function C(u,v) is known as a copula.

(iii) The Gumbel copula function is commonly used in insurance applications.

C(u,v) = exp[−((−ln u)a + (−ln v)a)1/a];a>=1

The Gumbel copula gives a strong tail correlation


.. and is also non symmetric/skewed, giving more weight to the right of the
distribution

This is useful for GI since, for example, large losses within a class and across
different lines of business are often strongly correlated or other relevant
example e.g. that it is the right of the tail that we need to model most
accurately for capital modelling.

Despite all being bookwork very few answered this question correctly, particularly the
formulae.

Page 4
Subject ST7 (General Insurance: Reserving and Practical Modelling Specialist Technical)
– Examiners’ Report – April 2012

2 (i) Changes in the (paid or incurred) loss ratio


Overall claims frequency
Frequency by claim type
..as some claim types (e.g. whiplash) may be more prone to fraudulent claims
or other relevant example
Number of claims arising from a single incident
Excessive claims cost inflation in particular areas
Increase in claims handling expenses
Nil settlements may indicate that claims are withdrawn after challenge
Benchmark or market data (analysis) or any other relevant external factor
Any other reasonable relevant factor e.g. average claim size, high renewal
rates, change in mix of business, low retention because fraudster changes
insurer to avoid detection

Generally reasonably answered.

(ii) Sharing data with other insurers to identify multiple claimants.


Invest in technology such as voice stress analysis systems to identify
fraudulent claims
Using software solutions to identify suspicious patterns of claims.
Increase the proportion of claims that are investigated prior to payment/
tightening of claims handling procedures/ specialist fraud unit
Publicity to highlight cases where policyholders prosecuted for fraud.
Enhance data analysis to provide indications of areas of
portfolio/policyholders more prone to fraud
..and either discontinue cover or increase premiums in these areas.
Education of policyholders e.g. if don’t disclose risk fully, claims will not be
paid
Encourage the public to report insurance fraudsters by setting up anonymous
phone lines etc to make reports.
Better staff training to help spot signs of fraudulent claims.
Lobbying government (directly or through industry bodies) for tougher
penalties for insurance fraudsters.
Fund police operations to target insurance fraud.
Tighten policy terms and conditions e.g. to demand a police report.
In time of recession/economic downtime fraud will increase and more work
needs to be done to combat it

In general candidates placed too much emphasis on increasing deductibles which would
deter most prospective customers, not just fraudsters.

3 Liabilities

General investment principle: to maximise investment return


… subject to meeting all contractual obligations
…and recognising the uncertainties involved.
The nature of the existing liabilities: whether they are they fixed or “real” in monetary
terms.

Page 5
Subject ST7 (General Insurance: Reserving and Practical Modelling Specialist Technical)
– Examiners’ Report – April 2012

The majority of general insurance liabilities will be real in nature.

Currency of existing liabilities:


many domestic, personal and commercial insurers may have portfolios mainly
denominated in their local currency,
however, international insurers and reinsurers have portfolios that contain a range of
currencies.

Term of existing liabilities:


most general insurers’ portfolios are likely to contain a significant proportion of short-
term liabilities (one–three years),
with a smaller proportion of medium term (four–ten years)
and long-term liabilities (ten years and above).

Whether the insurer has been subject to recent large losses


whether it is growing or shrinking
the position in the insurance cycle

Level of uncertainty of existing liabilities: both in amount and timing.


Reinsurance programme, backed up by reason/rationale

Estimated future liabilities arising from the portfolio of business planned: this will
depend on the volume and classes of future business written.

Assets may be required to be located locally to match local liabilities


Whether the liabilities are discounted.

Liquidity requirement
e.g. to pay claims as they arise
or for expected large losses

Need for diversification (should have rationale)

Assets

Size of assets, in relation to the current liabilities: the larger the quantity of free assets,
the more the company has freedom to invest widely.
And how much can move from principle of matching assets to liabilities by term,
currency etc.
Availability of assets

Expected long-term return from various asset classes.


Expected volatility within the various asset classes.
Existing asset portfolio.

Non-investible funds: not all the assets will be available for investment, for example,
moneys held by brokers, policyholders or reinsurers.

Page 6
Subject ST7 (General Insurance: Reserving and Practical Modelling Specialist Technical)
– Examiners’ Report – April 2012

Economic outlook.

External influences

Tax treatment of different investments and the tax position of the general insurer.

Statutory, legal, ethical or voluntary restrictions on how the insurer may invest.

Statutory valuation requirements.

Solvency requirements: most territories have a step-wise process of intervention and


therefore the strategy for maintaining solvency is targeted at a much higher level than
the pure statutory minimum margin.

Rating agency constraints on capital required to maintain the insurer’s desired rating.

Competition: strategy followed by other insurers.

Regulatory constraints, for example, those imposed by Lloyd’s and the Financial
Services Authority (FSA).

Insurer specific considerations

Risk appetite: each general insurer will differ in the amount of risk it is willing or able
to accept

Company specific investment objectives (for example, ethical investment).

Internal expertise

Risk analysis: market risk, credit risk, operational risk etc. (with rationale)

Usually well answered. Some candidates took the word constraints in its most literal sense of
limitations but Constraints is the word used as the heading for this bookwork in the Core
reading.

Page 7
Subject ST7 (General Insurance: Reserving and Practical Modelling Specialist Technical)
– Examiners’ Report – April 2012

4 (i) Balance sheet Assumptions

All yearly business


No reinsurance
Risks written uniformly across calendar year
Risk is uniform across policy year (or premium earned evenly over year)

Balance Sheet Assets A B

Investments 386 4,372


Current Assets 17 133
Deferred Acquisition Costs 29 128
Total 432 4,633

Liabilities

Outstanding Claims Reserve 94 3,076


Additional Unexpired Risk Reserve 47 0
Unearned Premium Reserve 76 556
Current Liabilities 35 171
Free Reserves 180 830
Total 432 4,633

This was a straightforward balance sheet question, but many could not handle it adequately.
Some put DAC on the wrong side of the balance sheet and some calculated the balance as
free reserves including share capital either as an Asset or as a Liability, and candidates often
included revenue items such as incurred claims or investment income in the balance sheet. A
minor point: even when getting the number right most gave DAC to 2 decimal places and
carried through in calculations, despite all data being given as whole numbers

(ii) Assumptions

Assume GWP = GEP (i.e. UPR b/f = UPR c/f)


Assume AURR as at 31/12/2010 = AURR as at 31/12/2011
Assume outstanding claims reserves include IBNR
Other reasonable assumptions are possible but then have to be carried through
in the calculations

A B
Incurred Claims 144 520
Acquisition Expenses 58 256
Total Expenses 75 385

Loss Ratio 95% 47%


Expense Ratio 49% 35%
Underwriting Ratio 144% 81%
Solvency Ratio 118% 75%
Return −57 275
Return on Capital −32% 33%

Page 8
Subject ST7 (General Insurance: Reserving and Practical Modelling Specialist Technical)
– Examiners’ Report – April 2012

Possibly more correct for Return on Capital to be based on Capital at start of


year: need to assume no dividend paid so Capital would be A: 237; B: 551 and
Return on Capital A: -24%; B: 50%.

A number of candidates took earned as half of written premium, even when assuming the
same premium written in 2011 as in 2010 but were not sensitive to the fact that this gave very
large loss ratios. Again straightforward, but many candidates were unable to complete this
question.

(iii) Comments

Company A may have suffered from adverse claims experience as shown by


its higher loss ratio compared to B.
Each company may be writing different classes or mix of business, each at a
different point in their respective market cycle
Company A expense ratio is higher due to higher acquisition expense ratio.
…The company is smaller than B and it may be spending money to expand
rapidly.
Company A solvency ratio is higher than B’s.
…This may be the result of a recent capital injection to expand the business.
Company B has the lower solvency ratio, suggesting that the company is less
financed than the other companies.
…Or it may have a stronger valuation basis for its assets/weaker for its
liabilities
Company B return on capital employed is the higher
supported by a larger relative investment return compared to A.
and by good underwriting results
The results given would change if reinsurance was considered
The expense ratio is high for each of A and B

In general this question was not well answered: too much emphasis was placed on the
obvious x>y etc., not enough thought was given to why the relationships existed.

5 (i) Advantages and Disadvantages

a: Underwriters view

Underwriter has knowledge of terms and conditions, market conditions etc.


Information from pricing review
Subjective
Potentially not independent
May be optimistic
Good when material changes to business

b: Market loss ratio benchmark

Simple to use

Page 9
Subject ST7 (General Insurance: Reserving and Practical Modelling Specialist Technical)
– Examiners’ Report – April 2012

Provides independent estimates


May reflect market rate changes and inflation effects as well as trends in
claims frequency and average cost
Mix of business likely to be different for every company
Need to check treatment of premiums is consistent (i.e. gross or net of
commission) and of reserving and other inconsistencies
Need to check consistency of reinsurance cover
Market info may not be available or may be out of date
If the benchmarks are up-to-date they must themselves be estimates and hence
have similar drawbacks to those we are facing in providing estimates.

c: Adjusted average over 3 years

Not independent
although expected loss ratio would be independent from claims on that year.
Can make correct allowance for rate changes and inflation
Reflects trends in own data
Low loss ratios indicate cat data so three year average loss ratio may not
reflect long term average
May not take account of changes in terms and conditions, mix of business,
underwriting cycle
Own data may be lacking or not credible but is large so should be credible

Any generic point such as cats probably not being allowed for under any of
these approaches

Generally well answered

(ii) Calculations:

1 . Roll forward 2007/8/9 LRs for Rate and Inflation


2. Average the 3 ratios
3. Use this ratio in BF calculation

BF Ultimate = Incurred Claims + IELR × (1 – inc dev factor) × Premium

U/wtg Year 2007 2008 2009


ULR% 24.60% 23.60% 27.70%
Inflation 1.1576 1.1025 1.0500
Rate Change 1.1130 1.1130 1.0500
Adjusted ULR% 25.59% 23.38% 27.70%

BF Ultimate = 9,000 + 0.2555 × 0.781 × 82,100


= 25,386
ULR = 25,386 / 82,100 = 30.9%

Page 10
Subject ST7 (General Insurance: Reserving and Practical Modelling Specialist Technical)
– Examiners’ Report – April 2012

As question is not specific could use premium weighted averages, also could
use ultimate premiums and ultimate claims rather than the ULR’s given:
answers are all similar and correct to at least 3 significant figures

Most candidates just calculated the adjusted average loss ratio and not the BF ultimate,
when the question specifically asks for the ultimate loss ratio with the average adjusted loss
ratio “for use within the Incurred Bornhuetter-Ferguson method”. Also some
overcomplicated things, for example by calculating indices or by working on incurred data
and then projecting. This level of complexity was clearly not needed with only 3 marks
available.

(iii) Reasonably high levels of downward development


Risk of being under-reserved if downward development does not materialise

May consider using paid development for 2006 to 2008 if danger of savings
not being produced but note that paid development is immature with percent
of ultimate varying from 77% to 50% across these years
But could consider paid BF for these years
And may consider incurred BF for 2009
But cannot use incurred BF for 2006 to 2008 without adjustment because
selected percentages of ultimate are greater than one
Ultimate loss ratios appear to be increasing and this needs to be investigated:
possibly should use trended loss ratio from years 2005 to 2009 to give a-priori
ratio for 2009: straight trend gives 28.3%
Should always use a number of different reserving methods before selecting
ultimates
If relevant such as considering completely different methods such as Cape
Cod, Mack etc. but probably not different forms of data such as accident year
or reporting year basis and probably not for suggesting chain ladder for
incurred claims as this is most likely to have been the method used to obtain
the selected incurred development

Very few candidates commented on the very low loss ratios and hence that this is likely to be
catastrophe-type business, in fact often stating that the loss ratio for 2009 is high. Some
suggested using chain-ladder as an alternative and hence did not realise that this is
presumably the method that the alternatives are required to be made to! Generally poorly
answered.

6 (i) Case estimates can make use of qualitative information on claims.


This can be important for unusual liability claims where data are scant.
Case estimates allow for the judgement of a skilled and experienced assessor

This can be particularly useful for large claims.


Reinsurance recoveries can be calculated explicitly using case estimated gross
claims.
Case assessors can assist in picking up trends more quickly than they emerge
using statistical methods.

Page 11
Subject ST7 (General Insurance: Reserving and Practical Modelling Specialist Technical)
– Examiners’ Report – April 2012

Case estimates may be the only available method when statistical methods are
unreliable
Such as in the early years of experience for liability and BI covers, when there
are insufficient data available.
Even when statistical methods are suitable, case estimates can still help to
review the appropriateness of the rating structure or a check on statistical
methods.
This is important for a new product.
Case estimates can be difficult to verify because they may be subjective.
Particularly for BI claims, where the financial state of the company may be
unclear.
And for long-tailed claims, where the rate of inflation assumed can be
significant.
Statistical methods are needed for estimating IBNR and reopened claims.
Case estimates are more resource-intensive than statistical methods.
Particularly for this product, where there is a wide range of different covers.
This makes it expensive and impractical to estimate small property and
liability claims individually.
Case estimates needed to be updated constantly as new information emerges.

Case estimates may have lack of consistency between assessors, due to


individual judgement or bias.
Case estimates may have lack of consistency over time.
Statistical methods allow the production of estimates on a range of different
bases.
The need to subdivide data for statistical methods to obtain homogeneous data
is a problem for this company as is small.
If a case estimate is used for negotiation with claimants, it may have a bias
towards the low end.
Advantages given for case estimates may validly be given as disadvantages for
statistical methods and vice versa

Many candidates gave a generic answer despite the question stating “for this product”.

(ii) Liability Covers


The claims will be for bodily injury and damage to/loss of property.
Many claims would be quickly notified & settled.
However, there can be significant delays in the emergence, reporting and
settlement of some claims.
This is particularly the case for larger claims and those involving bodily
injury.
Latent claims are a particular feature of Employers’ Liability.
There would be a wide distribution of claim costs and a high upper limit.
Claims inflation is an important factor for the long-tailed claims, particularly
court award inflation for bodily injury claims.
Accumulations can occur, due to:
…concentrations of risk from one event
…court awards triggering similar claims
…several insureds in same industry or type of trade being exposed to the same
cause

Page 12
Subject ST7 (General Insurance: Reserving and Practical Modelling Specialist Technical)
– Examiners’ Report – April 2012

Property Covers

The main types of claim would be for damage to business premises and stock
from perils such as fire, theft, water and weather.
Most claims occur suddenly and are quick to be notified, estimated and settled.

However, there are a few exceptions, such as notification of subsidence and


delays in verifying stock value (or other reasonable example).
Claims cost can be variable due to the variety of different types of property
and trade, but within each type the distribution of sizes tends to be consistent.

Accumulations can occur due to weather catastrophes.


There is an increased risk of fraudulent claims during an economic recession
where more businesses are in financial difficulties.

Business Interruption Covers

Claims are for financial losses following damage (e.g. fire) to business
premises.
Reporting delays are directly linked to any associated property claim.
However, settlement delays can be longer due to the need for additional
verification of the financial position and the time taken to restore the insured
to its pre-loss trading position (or other reasonable example)
Claim amounts can be very large due to the profits lost during a lengthy period
of restoration to the pre-loss trading position.
There is a risk of the insured attempting to inflate the claim amount above the
true loss, particularly in a recession where actual trading is depressed.

Many candidates outlined the different characteristics of each class but did not relate them to
the product and hence missed points such as that business interruption claims would be
related to property claim events.

(iii) Overall points

The choice of approach will depend on the reason for calculating the reserve.

...such as the granularity of reporting required (or other suitable example).


The choice of approach will depend on the statistical method to be used.
...because some methods may be more sensitive than others to stability of
development patterns (or other suitable factor).

Combining Property and BI

Advantages

Adds a greater weight of data in each cell, which should increase the stability
of the development patterns.
This is important for a new class of business where the company may not yet
have much data relating to particular types of cover or types of claim.

Page 13
Subject ST7 (General Insurance: Reserving and Practical Modelling Specialist Technical)
– Examiners’ Report – April 2012

It might be helpful for property and BI because claims will originate from the
same event.
It could reduce the uncertainty associated with the estimate of outstanding
claims.

Disadvantages

Development of claims costs could be distorted by the relative large size and
long settlement patterns of BI claims compared with property claims.
Trends for some types of claims may be difficult to spot if they are masked by
unusual experience for other types.
Loss adjustment expenses may be a very different proportion of the claims
cost for BI compared with property, which could distort the analysis.
Not all property claims have BI.

Splitting Liability

Advantages

Results in a more homogeneous data set, which reduces distortion from


different types of claims.
The allowance for future claims inflation can be more accurate (e.g., price
inflation for property; earnings inflation for liability).
The effect of economic conditions can be allowed for more accurately in
IBNR, since this will affect liability less than the other classes.
Liability itself should also be split if possible: PL different from EL.

Disadvantages

Data would probably be quite sparse for only 3 years of experience in a small
company, making estimation more difficult.
However, the property data would not have helped much with estimation of
liability claims anyway.
BF methods would become more difficult to use because the premium needs
to be split.

Given that there were six marks available for this section candidates generally gave too brief
an answer, missing many points.

(iv) Exposure based reserving may be suitable if statistical methods are unreliable,
such as following a one-off large loss (or latent claims)
…for example, a weather event, which would cause widespread losses in the
property and BI covers primarily (or other sensible example of severe loss).
…or where there is limited historical data, as is probably the case for this new
class.
It might not replace a statistical method entirely but would be useful as a
cross-check.
The method can take the form of a top-down analysis, under which a
proportion of the total market loss is attributed to the company.

Page 14
Subject ST7 (General Insurance: Reserving and Practical Modelling Specialist Technical)
– Examiners’ Report – April 2012

But this would not be appropriate here because of small size of company,
packaged product etc.
Alternatively, the company could take a bottom-up approach, which involves a
policy-by-policy analysis of exposure to the event.
This is method is more suitable where the likely exposure is atypical and the
exposure data are readily available and of adequate quality
…which is more likely in this situation.

Most candidates gave circumstances when exposure-based methods would be used and
described top-down and bottom-up approaches, but did not relate to the product described
and hence determine whether the methods were appropriate.

(v) Accounting rules may influence the basis, such as the level of prudence and
detail required for filing.
Timescales required for reporting results may influence the trade-off between
rigour and time taken.
The level of scrutiny (e.g. by auditors) may influence the level of rigour, such
as the number of cross-checks using different estimation methods.
Trading conditions in relevant industries may influence the level of IBNR in
anticipation of changes in claims activity.
Market rates of return will affect the discount rate.
Tax rules may impact the extent of discounting and margins or prudence.
Regulations for demonstrating solvency may prescribe a basis and method.
Professional guidance is likely to influence the methods and documentation
used.
Some methods such as BF may make allowance for position in underwriting
cycle
Competition (if reason given)
Trends in data because of the recession/propensity to claim
Reinsurance (if reason given)
For discounting, market rates of interest

The level of precision used will need to be proportionate to the quality of data
provided by the insureds.

Generally poorly answered.

7 (i) When would we use the capital model in the business

Regulatory requirements to satisfy “use test”:


that is, the company must use the model to help manage the business, not
simply to produce numbers for regulatory purposes.

Reinsurance: optimising the purchase of reinsurance


– types of RI and different retentions.

Page 15
Subject ST7 (General Insurance: Reserving and Practical Modelling Specialist Technical)
– Examiners’ Report – April 2012

Investment: assessing the impact of a change in the investment mix.


Investment departments often use model output to match liabilities in terms of
amount, timing, currency, etc.

Pricing:
assessing return on capital for pricing and performance measurement
used to form a view on how capital requirements should be allocated between
different lines on business.

Reserving: quantifying the uncertainty in claims reserves for regulatory


purposes

Planning:
Comparing different plans in terms of their risks
Comparing different plans in terms of their expected profits

Strategy:
Assessing the risks and diversification benefit of new strategies such as
consideration for writing new lines of business
Sensitivity testing of key assumptions such as exposure to CATS/inflation
shocks
Assessing the impact of using new distribution channels e.g. different credit
risk profiles/broker balances etc
Assessing the impact of introducing new Terms and Conditions to policies –
impact on pricing and capital can be assessed

Risk management:
identifying key risks and assessing the impact of mitigation.
Mergers and acquisitions

Generally poorly answered. All that is necessary is a slight amplification of a standard list
with some reasons.

(ii) Modelling UW and Reserving Risk for a new type of business

Underwriting risk

As a starting point, we can consider the firm’s business plan if this is prepared
on a realistic basis.
If the firm uses an aspirational business plan for motivational purposes, this
should first be adjusted to a best estimate basis.

For a new class we should be able to support the loss and expense assumptions
by reference to market experience, after adjusting for any differences.

The capital requirement for the underwriting risk is the difference between the
underwriting results at the firm’s chosen level of risk tolerance for the
business written/earned during the modelled period and the underwriting result
on the realistic basis.

Page 16
Subject ST7 (General Insurance: Reserving and Practical Modelling Specialist Technical)
– Examiners’ Report – April 2012

The realistic basis should not include any profit expected


(we should deduct such baseline profit from the capital requirement as a
separate item).

We should divide the firm’s business into classes/currencies/territories of


sufficient granularity (that is, small enough subdivisions) that we can consider
distinctive features of the class,
but not so fine that statistical methods become invalid (because of insufficient
data in the subdivisions).
Lack of data may inhibit this process

We should then assess the variability of its claims and expenses,


either by fitting statistical distributions
or by simpler approaches such as stress tests.

Ideally, we should model large claims separately from attritional claims so that
we can determine reinsurance recoveries directly.

However, for classes that are small or not subject to large claims it may be
more practical to model loss ratios.

We should model catastrophe-type claims separately from either,


especially for events that may impact more than one class.

We generally model large claims on a frequency/severity basis.

The Poisson distribution is often used for frequency,


but is only appropriate where the claims are independent,
since if there is any correlation between claims, this distribution will
underestimate the tail risk.

For severity, sampling from revalued past claim sizes is sometimes used,
but this omits the risk of a claim greater than experienced so it is preferable to
fit a distribution.

A heavily skewed distribution such as the Pareto would normally be


appropriate for severity,
Or a mildly-skewed distribution such as the lognormal may be appropriate
It should be derived from or tested against historic market data revalued to
current claims costs.
We generally model attritional claims in aggregate.

If the standard deviation is a sufficiently small fraction of the mean, a normal


distribution may be an adequate approximation.

We cannot model catastrophe events from the firm’s experience because of


their rarity and because a new business.
For natural catastrophes such as earthquake or windstorm, or for terrorist
attack, a proprietary model can apply a set of simulated events to the firm’s
exposure to derive a distribution of possible costs.

Page 17
Subject ST7 (General Insurance: Reserving and Practical Modelling Specialist Technical)
– Examiners’ Report – April 2012

Ensure that the model is suitable; for example, by allowing for demand surge,
climate cycle, and so on,
and to test the results against the known impact of recent actual catastrophes
and to resolve or adjust for any discrepancy.
For human-made catastrophes other than terrorism, the firm is likely to have to
develop a bespoke model.

Gross reserving risk

New risk so potentially no immediate impact.

Firm could make allowance for 2 years of hypothetical reserves to avoid future
capital strain. e.g. Lloyds

Lack of data and experience an issue so industry data needed – and associated
problems

We should then assess the variability in the firm’s claims settling, by statistical
techniques such as bootstrapping or the Mack method, or stress tests.

New class/industry data so need to consider whether sufficient reserve shocks


have occurred within the period of the reserve data to indicate possible future
variability.

Possibly adopt a greater variability than the industry figure,


or we could model an explicit shock such as a future Ogden rate change.

To convert the capital impact of the gross insurance risk to that for net
insurance risk within a class, we should deduct the reinsurance that can
contractually be recovered on large and catastrophe claims
(and on attritional claims in the case of proportional covers and working
layers).

We should allow for disputes or exhaustion of cover,


so it is preferable to calculate reinsurance recoveries directly, rather than
assume that historic net to gross ratios will continue in the future
Need to make an assessment of the correlations and dependencies between
classes/perils etc. based on historical and market data

Many candidates did not consider what was being parameterised e.g. splitting claims into
attritional, large and cat claims and the parameters required for each of these.

(iii) Potential impact of inappropriate model

• It could lead to the incorrect decision on whether or not to start writing this
line of business
• which has subsequent consequences on future profitability

Page 18
Subject ST7 (General Insurance: Reserving and Practical Modelling Specialist Technical)
– Examiners’ Report – April 2012

• impacts all stakeholders including:


o shareholders (returns),
o employees (job security),
o policyholders (claim payments)
o Other stakeholders of the company (e.g. suppliers, competitors).

• If reserve risk parameters/model incorrect, all estimates of the reserves in


the model (gross, net, undiscounted, discounted), at each point in time they
are modelled (e.g. opening, end of year 1, etc.) will be unreliable both for
the individual class and in total.

• If UW risk parameters/model incorrect, all estimates of profitability of


business written in future years in the model will be unreliable both for
individual class and in total.

• Because the projected balance sheet is unreliable, any view on capital


requirements or surplus derived from this is also unreliable.

• The company is at risk of holding inadequate capital for its true exposures
should it start writing the new class as it has an unreliable view on its
liabilities. This poses direct threat to the security of policyholders.

• If errors lead to too much capital there is an opportunity cost representing


the use to which the capital could otherwise have been put

• There are reputation risks to the company if it turns out that inadequate
capital is held. This has second order impact on ability to write future
business, borrow money, etc.

• Pricing: With incorrect allocations to classes of business the company is


writing business at inadequate rate which causes further profitability
issues.

• incorrect performance management results will be made based on


inadequate model
• Investment management: If the payment pattern parameterisation is
incorrect it is possible the selected asset strategy will be accordingly
incorrect.

• Risk management If, the correlation assumptions in the model are


incorrect, the insurer may think it has more diversification in its portfolio
than it actually has.

• If the company models future premium growth incorrectly, it may not


understand how its exposure to catastrophes or class specific inflation
changes over time.

Page 19
Subject ST7 (General Insurance: Reserving and Practical Modelling Specialist Technical)
– Examiners’ Report – April 2012

• It is possible that the incorrect decision will be made in reinsurance


purchases, which in turn will affect future profitability and exposure to
risk.

• Has an impact on any further operational planning or M&A analysis


carried out using the model, the results of which may be misleading if say
the premium correlation assumptions are incorrect.

Generally poorly answered, with most candidates finding very few points on this part.

END OF EXAMINERS’ REPORT

Page 20
INSTITUTE AND FACULTY OF ACTUARIES

EXAMINATION

24 September 2012 (pm)

Subject ST7 – General Insurance:


Reserving and Capital Modelling
Specialist Technical

Time allowed: Three hours

INSTRUCTIONS TO THE CANDIDATE

1. Enter all the candidate and examination details as requested on the front of your answer
booklet.

2. You have 15 minutes at the start of the examination in which to read the questions.
You are strongly encouraged to use this time for reading only, but notes may be made.
You then have three hours to complete the paper.

3. You must not start writing your answers in the booklet until instructed to do so by the
supervisor.

4. Mark allocations are shown in brackets.

5. Attempt all eight questions, beginning your answer to each question on a separate
sheet.

6. Candidates should show calculations where this is appropriate.

AT THE END OF THE EXAMINATION

Hand in BOTH your answer booklet, with any additional sheets firmly attached, and this
question paper.

In addition to this paper you should have available the 2002 edition of the Formulae
and Tables and your own electronic calculator from the approved list.

ST7 S2012 © Institute and Faculty of Actuaries


1 List, giving examples, sources of uncertainty in modelling future expense levels. [7]

2 Explain when it would be suitable to use the Bornhuetter-Ferguson method to


estimate ultimate claim costs and any problems with its use. [4]

3 List the disadvantages associated with the restrictions and regulations that a
regulatory authority might put in place. [4]

4 (i) Give examples of accumulations of risk that could arise from writing the
following classes of business:

(a) Motor fleet


(b) Professional indemnity
[3]

(iii) Suggest methods of mitigating accumulation risk for the above classes. [3]
[Total 6]

5 (i) Describe components of market risk that an insurer should consider when
developing a capital model. [5]

A personal lines insurance company writes motor, household and creditor insurance
business in two different territories.

(ii) Describe potential sources of correlation between these lines of business. [5]

(iii) Describe potential sources of correlation between insurance and market risk
for each of these lines of business. [5]

(iv) Give examples of applications of the output that could demonstrate that a
capital model satisfies a “use test”. [5]
[Total 20]

ST7 S2012–2
6 (i) List the criteria that risk and risk events should satisfy to be insurable. [4]

A general insurance company is considering the insurability of the following two


proposed insurance or reinsurance arrangements, details of which are summarised
below.

Proposal A: Insurance for a school fundraising game

• The school requires insurance cover for a game to be held at their annual
fundraising event. The game involves a competitor paying an entrance fee of £10
to roll six dice. If he rolls six sixes, he wins a prize of £25,000. If he rolls any
other combination, he wins nothing.
• The school wishes the insurance cover to indemnify it for the £25,000 prize if
there are any winners.

• The head teacher is confident that there will be at least 100 competitors on the
day.

Proposal B: Utility price increase cover

• The product development department has proposed a new type of insurance


cover.. Policyholders will be protected in the event of significantly increased
household bills for electricity, gas and water.

• The insurer will pay policyholders if increases in any of their monthly bills exceed
the current monthly rate of retail price inflation. The insurer will pay the excess
over that level up to a specified limit.

(ii) Discuss the extent to which each of Proposals A and B satisfies each of the
criteria in (i). [6]

(iii) Suggest amendments to each proposal to improve the insurability of the risks.
[4]
[Total 14]

ST7 S2012–3 PLEASE TURN OVER


7 (i) Outline the following approaches for allowing for outwards reinsurance in
reserving calculations, stating the advantages and disadvantages of each:

(a) Separate projections of gross and net of reinsurance triangles.


(b) Using net/gross ratios.
[9]

The following table is an extract of information relating to one class of business of a


large insurer. The data is as at 30 June 2012.

Underwriting Net/Gross Ratios


Year Premium Paid claims Outstanding claims

2007 60% 70% 70%


2008 62% 80% 70%
2009 55% 45% 10%
2010 80% 90% 65%
2011 70% 100% 95%
2012 −14% 100% 100%

A full gross of reinsurance reserving analysis is also available.

(ii) Explain how this information can be used to estimate net of reinsurance IBNR.
[3]

(iii) Suggest a reason why the net/gross premium ratio is negative for the 2012
underwriting year. [1]

The only reinsurance programme in place for the 2005 underwriting year was a 20%
quota share.

The 2006 reinsurance programme comprised a 25% quota share with a profit
commission that was payable by the reinsurer defined as 20% of the difference
between the gross loss ratio and 70% when the gross loss ratio is less than 70%. The
current expected gross loss ratio for 2006 is 50%.

(iv) Calculate the three net-to-gross ratios for the 2005 and 2006 underwriting
years, stating any assumptions. [3]

The company is attempting to reduce its future reinsurance cost and is considering
purchasing, for the 2013 underwriting year, excess of loss cover with significant
reinstatement costs, rather than free reinstatements as is currently the case.

(v) Describe how the net/gross ratio analysis could be amended to allow for
reinstatement premiums. [3]

(vi) Explain whether it would be appropriate to use net/gross ratios to transform a


gross-of-reinsurance reserve range into a net-of-reinsurance range, suggesting
an alternative approach. [4]
[Total 23]

ST7 S2012–4
8 The following data have been supplied to a consulting actuary as part of a peer review
of the projections made by the reserving department of a general insurance company
using an automated chain ladder programme.

Claims Paid:

Development Year

Underwriting Ultimate Future


Year 1 2 3 4 5 Claims Claims

2007 3,198 4,718 5,641 6,112 6,125 6,125 0


2008 3,405 5,111 6,140 6,610 6,624 14
2009 3,395 5,196 6,182 6,690 508
2010 3,723 5,798 7,501 1,703
2011 3,803 7,467 3,664
5,889

Development Factors:

1–2 2–3 3–4 4–5 5–ult

Year-on-Year 1.5176 1.1955 1.0799 1.0021 1.0000


Cumulative 1.9634 1.2938 1.0822 1.0021 1.0000

(i) Outline the changes that the consulting actuary might make to the chain ladder
development factors used and hence recalculate future claims. [8]

The company has also supplied the following data for a class for which development
data are only available as at the dates given and has requested advice on how to adapt
the data for use in the chain ladder method.

Cumulative Claims Paid as at


Underwriting 30/11/07 31/05/09 31/12/09 31/03/11 31/12/11
Year

2007 1,757 3,267 3,595 3,936 3,940


2008 2,502 3,187 3,889 4,201
2009 950 2,405 4,061 4,578
2010 3,039 3,865
2011 307 2,810

(ii) Construct a standard annualised triangle of claims data and hence calculate
future claims as at 31 December 2011, making any adjustments that you
consider necessary and stating any assumptions made. [14]
[Total 22]

END OF PAPER

ST7 S2012–5
INSTITUTE AND FACULTY OF ACTUARIES

EXAMINERS’ REPORT
September 2012 examinations

Subject ST7 – General Insurance:


Reserving and Capital Modelling
Specialist Technical

Introduction

The Examiners’ Report is written by the Principal Examiner with the aim of helping
candidates, both those who are sitting the examination for the first time and using past papers
as a revision aid and also those who have previously failed the subject.

The Examiners are charged by Council with examining the published syllabus. The
Examiners have access to the Core Reading, which is designed to interpret the syllabus, and
will generally base questions around it but are not required to examine the content of Core
Reading specifically or exclusively.

For numerical questions the Examiners’ preferred approach to the solution is reproduced in
this report; other valid approaches are given appropriate credit. For essay-style questions,
particularly the open-ended questions in the later subjects, the report may contain more points
than the Examiners will expect from a solution that scores full marks.

D C Bowie
Chairman of the Board of Examiners

December 2012

© Institute and Faculty of Actuaries


Subject ST7 (General Insurance: Reserving and Practical Modelling Specialist Technical) –
September 2012 – Examiners' Report

General comments on Subject ST7

Candidates who are well prepared generally appear to perform reasonably on ST7, with the
more challenging questions tending to occur on SA3. Candidates should consider the
following advice however (if they are not already):

• Lists are hugely valuable for breadth of point generation but candidates should always
exercise judgement when applying them.

• Calculation questions will come up on a regular basis with ST7, as candidates can clearly
observe from examination of historical papers. Candidates should always be prepared for
such staples as balance sheet preparation, triangle manipulations & projections and
reinsurance layer calculations (along with being able to carry out any necessary
adjustments including inflation, exposure and time period issues).

• Capital questions should be expected on every paper and represent a sufficient proportion
of the course content that candidates should not expect to be able to pass on their
reserving knowledge alone. Those who do not encounter capital work in their
professional lives should be particularly careful to ensure that they take time to
familiarise themselves with this element of the course.

• Candidates should aim to be able to give near exact glossary definitions as incoherent or
vague descriptions will be marked harshly. If candidates struggle to remember definitions
verbatim they should take the time to properly analyse the glossary definition to ensure
they have fully absorbed all the nuances of the definition.

• It is important to always read the question properly.

Comments on the September 2012 paper

There were a number of disappointing areas on this paper, with failing candidates simply not
displaying any capacity for independent thinking on question specifics. Depth was often
lacking, with many candidates creating general lists for some questions rather than applying
to the question specifics, although this may reflect a failure to read the question properly.
Comments can be found under each question, but the most concerning areas of weakness
were:

• Q4 – many candidates struggled with the mitigation question, displaying an excessive


tendency to focus on cover changes that would clearly be at best inappropriate and at
worst illegal for compulsory covers; these candidates were fortunate that the marking
system does not allow negative marks to be given. These candidates also generally failed
to consider any of the wider engagement that a company could have with coverholders or
any of the internal exposure management processes that could mitigate accumulation
risks.

• Q5 – typically for capital questions, this was poorly answered. While many candidates
clearly have no direct professional experience with capital modelling, broad knowledge of
general insurance and product lines should have been sufficient to e.g. identify possible
sources of correlation.

Page 2
Subject ST7 (General Insurance: Reserving and Practical Modelling Specialist Technical) –
September 2012 – Marking Schedule

• Q6 – many candidates seemed to have an incredibly superficial understanding of key


concepts which fell over when they were asked to apply those concepts. For example,
many candidates thought that there was no insurable interest seemingly because the
proposal was to fully insure the losses.

• Q7 – this was the worst performing question. Candidates were generally unable to
consider any nuances of Net to Gross ratios and showed flawed understanding of the
mechanics of reinsurance contracts. Most disappointingly, only a small minority of
candidates were aware that net to gross premium ratios are often negative for very
immature years due to timing differences.

• Q8 – this was broadly reasonably answered, but a significant number of candidates did
not seem to pick up on the very clear instruction to “recalculate”, sacrificing a number of
marks that were generally high scoring for the candidates who had read the question.

Page 3
Subject ST7 (General Insurance: Reserving and Practical Modelling Specialist Technical) –
September 2012 – Examiners' Report

1 Expenses uncertainties:

Broker mergers/ change in mix of brokers


. . . lead to changes in commission payments
New distribution channels / markets / lines of business
. . . . costs of these may not be fully understood
Aggregators
. . . Various examples, e.g. may have entry criteria or different commission structures
such as a per policy charge
Accounting changes
. . . . Can lead to additional costs for training / consultants
Offshoring/outsourcing
. . . . can have high set up & redundancy costs & uncertain savings
Taxes / levies / regulatory
. . . . any example of changes to these factor
Economic factors
. . . any relevant example e.g. inflation / RPI / changes in rates of exchange
M&A activity
Other strategic decisions e.g. aggressive marketing campaign
Number of policies/volume of business
Changes in levels of claim/ reinstatement costs
Management fees
Changes in mix of business
Profit commission to brokers/profit related pay
Changes in pension arrangements
Lack of data
Any other reasonable sources

Generally well answered. Poorer answers simply tended to get fewer of the points than good
answers. Occasionally people answered about what expenses might arise rather than
sources of uncertainty and a few went off into answers about model uncertainty in general.

2 When to use the Bornhuetter-Ferguson method

The Bornhuetter-Ferguson method is very useful where the available data for the
particular cohort are sparse.

This is often the case with more recent cohorts, cohorts from longer tailed portfolios
(for example, liability excess of loss reinsurance) or where claims activity is expected
to be extremely volatile.

It can also be used when a blend of experience and an exposure based estimate is
deemed appropriate.

The Bornhuetter-Ferguson is most effective when the current data are too immature to
be developed on a projection method,
… but we believe the experience data to date still gives some indication of the level of
ultimate claims.

Page 4
Subject ST7 (General Insurance: Reserving and Practical Modelling Specialist Technical) –
September 2012 – Marking Schedule

Problems with using the Bornhuetter-Ferguson method

More complex to apply and explain


Need reliable source of exposure-based assumption, e.g. IELR

In some cases, the data being projected may develop with a negative tail. In these
situations, we may consider the assumed percentage developed unsuitable as a weight
in this credibility method.

…We may use different weights, or where the incurred development displays a
negative tail, we may consider it is more appropriate to apply the Bornhuetter-
Ferguson method to the paid development.

In practice it would be unusual to use the Bornhuetter-Ferguson method after the first
few development years. A chain ladder or case estimate approach would be preferred
at longer durations.
Any generic problems applicable to all reserving methods, e.g. large claims

Reasonably answered. Most candidates got the most important points but few gave enough
to get full marks.

3 Possible disadvantages of regulation:

The cost in terms of resource and finance to comply with, supervise and interpret the
rules
,,, in particular for changes to the rules
The loss of business opportunities that arise from any restraint on a free market e.g.
minimum/maximum premium rates.
The inability to maximise investment returns when there are controls on the
investment decision.
The quantum of regulatory bureaucracy deterring new entrants.
The difficulties and hence potential inaccuracies in complying with complex (risk-
based) liability and capital calculations.
The increased premium cost to the public arising from levies and the general increase
in insurer expenses.
The inability of companies to benefit from economies of scale and cost reduction due
to anti-competitive legislation.
The failure of insurance to reach certain sectors of the population due to the increased
cost of and restrictions on methods of distribution.
Inflexibility of rules: one size fits all
Check list mentality/ false sense of security if all boxes are ticked
Rules may act against the principles of insurance: e.g. no sex discrimination on
premium rates which also increases overall premium rates
Any other reasonable disadvantage

Probably the best-answered question with quite a few full marks. Some candidates just listed
restrictions and regulations without saying why they were disadvantageous.

Page 5
Subject ST7 (General Insurance: Reserving and Practical Modelling Specialist Technical) –
September 2012 – Examiners' Report

4 (i) Macroeconomics/ changes in legislation

(a) Fleets may be parked or based in a similar geographical area.


And therefore prone to multiple claims from natural catastrophes e.g.
storm/flood/hail damage.
Or a single large loss such as an industrial estate explosion or fire.
Risk management policies of large fleets e.g. drivers not taking regular
breaks

(b) Exposure to a single profession can give rise to aggregations


e.g. property surveyors at a time of falling prices
Systemic bad advice/ control failings
Any other valid point under (a) or (b)

(ii) Encourage and support insurance risk management


Carefully and regularly monitor aggregations of risk by geography/profession
in management information.
..supplemented by purchasing specialist software/data such as that which
models flood exposure
..and when defined limits are reached stop underwriting.
May need to non-renew or significantly increase rates if current aggregations
too high
Purchase additional reinsurance that protects against aggregate events
Or enter into a quota share arrangement to reduce overall exposure
Consider amending terms, conditions and exclusions e.g. reducing policy
limits for certain events
Increase diversification by writing more classes, or new geographical areas
including overseas, new professions.
Keep fleet cars at drivers’ homes
Any other reasonable method

Part (i) – Moderately answered. A lot of candidates gave poor examples of accumulations
such as the possibility of motorway pile-ups involving more than one car in a fleet. In (b) too
few got the point about concentrations arising from specialisation in a particular profession
with a systemic problem.

Part (ii) – Too few got the points about monitoring concentrations so they do not arise or
encouraging good risk management. Too many suggested inappropriate cover changes, such
as limits on claims that would be illegal in the case of motor or changes that would make the
insurance very unattractive, such as disallowing claims that arose from concentration. Most
candidates got the points about diversification. Some candidates did not mention
reinsurance whereas some others concentrated wrongly on all types of reinsurance.

Page 6
Subject ST7 (General Insurance: Reserving and Practical Modelling Specialist Technical) –
September 2012 – Marking Schedule

5 (i) Equity risk


the risk that there is a significant reduction in the value of an insurers equity
portfolio
(including dividend income)

Property risk
the risk that there is a significant reduction in the value of an insurers property
portfolio
(including rental income)

Foreign Exchange risk


the risk that exchange rates move in an adverse manner, reducing the value of
the insurer’s assets.

Interest Rate risk


The risk that interest rates move in an adverse direction leading to a reduction
in the value of bond portfolio

Spread risk
The risk arising from the change in the relationship between interest rates in
different market sectors.

Concentration risk
The risk from holding bonds or other investments from a limited number of
counterparties.

Mismatching risk
The risk from holding investments that do not match expected liabilities by
amount, timing etc.
Also:
Counterparty/issuer defaults
Severe economic or market downturn or upturn leading to adverse interest
movements and/or equity market falls
Inadequate valuation of assets
Liquidity risk that cash is not available when claims need to be paid
Any other reasonable risk

(ii) Motor and household

Claims inflation may be higher in both classes in times of high inflation


Large single events may cause losses in both (e.g. residential fire, damaging
nearby vehicles)
Weather events and natural catastrophes may cause damage to both (e.g.
floods, earthquakes, hailstorms)

Household/motor and creditor


The insurance cycle may reduce premium adequacy in more than one class at
the same time (e.g. competitive new entrant into both markets)

Page 7
Subject ST7 (General Insurance: Reserving and Practical Modelling Specialist Technical) –
September 2012 – Examiners' Report

Economic environment
E.g. increased unemployment may lead to both increased creditor claims
and increased theft / fraud in the household and/or motor books

Operational issues within the company may impact more than one class e.g:

• Systemic poor underwriting


• Systemic poor claims control e.g. due to staff stretch
• Management strain leading to poor oversight
• Poor strategy at company level

Other external factors:


• Legislative environment
• Propensity to claim
Any other reasonable potential source of correlation

(iii) A recession could lead to losses on the company’s investments


..and unemployment that will cause more creditor claims and crime/fraud for
household/motor.
A significant fall in property values could lead to investment losses
..and increased arson/other fraudulent claims on property policies.
A particularly large catastrophe could trigger market falls.
A significant change in exchange rates could impact the value of reserves held
..particularly the case if assets and insurance liabilities are not matched.
The company may have investments in companies whose collapse could lead
to insurance as well as investment losses
..for example if creditor policies are offered to employees of a particular large
company.
Mis-valuation of assets could also mean mis-valuation of the reserves
...perhaps due to systematic control and governance problems in the company.
Inflation correlated with interest rates
Any other reasonable potential source of correlation

(iv) Setting regulatory capital.


Reinsurance: optimising the purchase of reinsurance by testing alternative
structures.
Assessing profitability of new lines of business/existing business is evaluated
using the internal model
Projecting future profit and loss accounts enabling testing of actual experience
compared to expectations.
Informing, managing or reviewing risk appetite
Reviewing investment portfolio and testing alternative strategies.
Regular and evidenced Board review of internal model output.
Capital allocation to individual underwriting units.
Aggregation monitoring/ assessing catastrophe exposures.
Designing and monitoring risk management systems: identifying key risks and
assessing the impact of mitigation
Setting bonuses/performance related salary for underwriters/management
Pricing: assessing return on capital for pricing and performance measurement

Page 8
Subject ST7 (General Insurance: Reserving and Practical Modelling Specialist Technical) –
September 2012 – Marking Schedule

Reserving: quantifying the uncertainty in claims reserves


Planning: assessing different plans in terms of their risks, not just expected
profits
Strategy: assessing the risks and diversification benefits of new strategies
Any other reasonable application

Part (i) – Some good answers, and others where the main fault was not covering enough of
the points. Too few adopted the structure of naming the risk then explaining what it was.
Too many concentrated on general economic factors rather than the components of market
risk. A few went off on the wrong track, e.g. writing about how to model risks.

Part (ii) – Not generally well answered. A significant number did not understand what
creditor insurance is. Quite a few failed to distinguish between the three classes in respect of
inflation, which would not increase creditor claims. Few candidates got the points about
general company issues affecting all classes.

Part (iii ) – Very poorly answered. Most candidates made only one or two points and these
were often confused. Too often they gave sources of correlation within each of the two
categories, insurance and market risk, rather than between the two categories.

Part (iv) – Reasonably well answered. Most candidates got a good number of points and
there were a few full marks. Occasional scattergun approaches listed a few things an
internal model cannot do.

6 (i) 1. The policyholder must have an interest in the risk being insured, to
distinguish between insurance and gambling.
2. A risk must be of a financial and reasonably quantifiable nature.
3. The amount payable by the insurance policy in the event of a claim must
bear some relationship to the financial loss incurred.
4. Individual risk events should be independent of each other.
5. The probability of the event should be relatively small. In other words, an
event that is nearly certain to occur is not conducive to insurance.
6. Large numbers of similar risks should be pooled to reduce the variance
and hence achieve more certainty.
7. There should be an overall limit on the liability undertaken by the insurer.
8. Moral hazards and possibility of should be eliminated as far as possible
because these are difficult to quantify, result in selection against the
insurer and lead to unfairness in treatment between one policyholder and
another (loss events should not be under the control of the insured)
9. There should be sufficient existing statistical data/information to enable
the insurer to estimate the extent of the risk and its likelihood of
occurrence.

(ii) Proposal A

1. The school does have an interest in the risk because they will have to pay
if someone rolls six sixes.
2. The risk is easily quantified (as long as the dice are not biased).

Page 9
Subject ST7 (General Insurance: Reserving and Practical Modelling Specialist Technical) –
September 2012 – Examiners' Report

3. The amount paid by the insurance company is linearly related to the


amount raised or lost from the event.
4. Each set of dice rolls is independent from the others.
5. Probability of each risk event is small (1 in 66 = 1 in 46656).
6. 100 competitors may not be considered a large enough number of risks to
reduce variance and highly unlikely to get even 100 with £10 fee or
insufficient income for an insurer to be interested.
7. There is no limit to the payout as several competitors might win although
highly unlikely (if no fraud).
8. This criterion is not easily satisfied as there would be potential for
corruption.
9. The basic probabilities are easily computed but existing data as regards the
profitability of the event (which might include a factor for moral hazard)
may not be available.

Proposal B

1. Policyholder does have an interest in the risk.


2. Risk is financial and quantifiable.
3. The amount payable linearly related to bill.
4. There may be some independence between gas, electricity and water
prices, although there is likely to be correlation between gas and electricity
as they are both energy related, but different policyholders’ bills for each
utility are likely to be highly correlated.
5. Probability of price increases exceeding RPI is quite high.
6. Likely to be a large number of risks but no reduced variability as per
point 4.
7. Possible for there to be claims from all policyholders.
8. Moral hazard could occur in that the policyholders use more electricity,
gas or water.
9. Probably possible to analyse past gas, electricity and water rate increases
for different suppliers but more likely to rely on fundamentals.

(iii) Proposal A
Reduce entrance fee to £1 and the prize to £2500 (for example) (or get
sponsorship from a car dealer for a car as the prize), which is likely to result in
a larger number of individual attempts and thus a greater pooling of risk, and
reduction in variance
Increase the number of dice from 6 to 7 to reduce the probability of a claim
Limit the cover to the first win or a fixed number of wins only.
Shared prize if more than one winner
Protect against moral hazard/fraud by having an adjudicator on site or
videoing the whole event, having the insurer supply the dice and having robust
exclusions for negligence/fraud.
Using one of the very few insurance companies that write this form of
insurance and who therefore have a large pool of such risks
Any other reasonable amendment

Page 10
Subject ST7 (General Insurance: Reserving and Practical Modelling Specialist Technical) –
September 2012 – Marking Schedule

Proposal B
Specify that the cover applies to increases in unit rate not overall cost of
consumption.
Limit maximum payout per policyholder.
Limit cover to those buying from particular companies
Change the trigger for the cover from the current rate of RPI to some adjusted
alternative, such as allowing for historic utility inflation being higher, to
reduce likelihood of a claim.
Any other reasonable amendment

Part (i) – Generally very well answered with quite a few full marks.

Part (ii) – Generally well answered for (a), with a few problems. A few did not read the
question properly assuming that the number of dice thrown was 1, 2 or 3 which was not
sensible given the monetary amounts involved. A large number said that the school did not
have an interest in the insured event, even when they stated in (i) that this was to distinguish
insurance from gambling, because if it was insured they made no loss if someone won.
Candidates also often stated that there was no moral hazard which it is clear there is as if the
event is insured then the school could well act differently. A lot of candidates said that the
attempts were independent unless the dice were biased which is not true as so long as the
same dice are used: biased dice have the wrong probability of success but the same
probability for each attempt and every attempt is independent. The notion of pooling was
somewhat confused: pooling will exist if the insurer can find a lot of independent events to
insure, which may not be identical but that will have low probabilities of big payoffs. The
point about a maximum loss was also somewhat confused; clearly it is £25,000 times the
number of entries but that is a very big loss and might be considered extreme, if not
unlimited, although with low probability. (b) was reasonably answered, with most
candidates realising that events were not independent, although it was not always well
expressed. A very good number made the point about moral hazard and over-using utilities.

Part (iii) – The main fault here was brevity. Most candidates made one or two good points
and left it there. A surprising number suggested limiting the number of entrants in (a)
although the question does not state that the premium would be fixed. There were some
impractical suggestions.

7 (i) (a) Reserving using data gross and net of reinsurance

Reserve as normal on a gross basis and then apply same techniques to


net triangles

+ves

Can compare the resulting gross and net projections with the difference
being the reserves for reinsurance.
Simple to apply and understand
Simple to add to semi-automated reserving process
Can be used to assess the volatility of the net outcomes

Page 11
Subject ST7 (General Insurance: Reserving and Practical Modelling Specialist Technical) –
September 2012 – Examiners' Report

Appropriate for proportional covers and low working layers (and,


according to core reading, for very high excess reinsurance)
Appropriate where reinsurance programme has been stable over a
number of years
The method can be adjusted to allow for major catastrophes
{Credit given if the negative statement is given as a disadvantage}

-ves

Possibility of inconsistent (including negative) reinsurance recoveries


Full historic triangulations of net data may not be available at the
equivalent granularity
..particularly if there are whole account or grouped lines of business
covers
Lack of direct link between gross and net experience could lead to
inconsistent results for capital/ERM purposes.
Does not permit accurate assessment of credit risk
Other reasonable valid advantages/disadvantages

(b) Applying net/gross ratios to gross reserves

Derive a net to gross ratio using historic experience and details of RI


programme
...then multiply ratio by gross reserves to obtain net reserves or by
gross ultimates to obtain net ultimates depending on which is
considered appropriate.
… or, rarely, can calculate gross from net by dividing by the ratio

+ves

Simple to apply, particularly for proportional covers


No need for full triangulations of net data – just latest positions
Ensures consistency of gross and net results (no negative recoveries)

-ves

May be difficult to derive a single ratio that is appropriate for a


particular class of business.
..particularly when complex (non proportional) features to reinsurance
programme.
Other reasonable valid advantages/disadvantages

Under (a) or (b):

Cannot accurately allow for some features of reinsurance such as


aggregate limits, aggregate retentions and profit commissions.
Cannot accurately allow for claims that exhaust vertical cover.
Other reasonable valid advantages/disadvantages

Page 12
Subject ST7 (General Insurance: Reserving and Practical Modelling Specialist Technical) –
September 2012 – Marking Schedule

(ii) A net/gross ratio can be derived that is appropriate to apply to the gross IBNR
to obtain an estimate of the net IBNR.
Key point: the selected IBNR ratio may be based on a different financial for
each underwriting year based on the recoverability that is most similar.
For the oldest underwriting years, outstanding may be most appropriate given
small number of claims remain or IBNR may be taken as zero
(Comment: here and following probably you should give suggestions as to
which years from the table are appropriate)
For more recent years, incurred claims may be most appropriate.
For the newest underwriting years, use net/gross premium ratio (if reasonable)
as unlikely to be enough claims notified to be stable.
… although as a large insurer there may be sufficient claims
But cannot use premium ratio for 2011 as negative: may need to obtain from
other source such as business plan but not given in question data
Alternatively, averages of ratios may be used.
Adjusting the data for any known catastrophes or unusual events
And allowing for changes to the reinsurance cover (e.g. commuted covers).
Alternative approach may be reasonable

(iii) A negative net/gross ratio implies that the reinsurance cost is greater than
gross premiums received..
..often due to the reinsurance having to be paid for in advance of premium
receipt e.g. as minimum and deposit premium

(iv) 2005
n/g premium 80% n/g paid 80% n/g outstanding 80%
2006
n/g paid 75% n/g outstanding 75%

Profit commission receivable : (70% - 50%) × 0.2 = 4%


Therefore n/g premium 75% + 25% × 4% = 76%

Assumptions
Figures quoted are net of ceding commission
Profit commission payable as a premium refund rather than claims deduction

Commission calculated as percentage of reinsurer’s premium


Actual gross loss ratio equal to expected
Note that alternative answers may be applicable if the assumptions given are
reasonable e.g. 75%, 75%, 79% or 74%, 74%, 75% or 71%, 71%, 75% or
75%, 75%, 75% the latter only if the profit commission is calculated and
stated to be included in a net to gross analysis as a separate item

(v) Reinstatement premiums will increase the cost of reinsurance programme in


the event of losses..
...therefore would expect net/gross premium ratio to be lower.
For known losses, would be able to calculate the reinstatements exactly and
add to the reinsurance cost.

Page 13
Subject ST7 (General Insurance: Reserving and Practical Modelling Specialist Technical) –
September 2012 – Examiners' Report

For future losses, should consider the reinsurance IBNR that has been
estimated
..and use the reinstatement terms to estimate the cost that relates to that
recovery.
Some approximation may be required to allocate RI IBNR between different
layers/programmes.
Alternatively could use a simulation model that could estimate mean
reinstatement costs based on a distribution of gross losses.
Any other reasonable valid points/(interpretation)

(vi) Applying a net to gross ratio may give some indication of a net range
..but is likely to be too crude to give a reasonable range, particularly towards
the tails of the distribution
May be higher recovery rate at higher percentiles as retention eroded on any
aggregate excess of loss reinsurance protection so additional gross losses are
fully recoverable.
..or lower if significant reinsurer default due to a large market event
..or through top of programme
Therefore deriving a distribution of net/gross ratios to apply at different
percentiles could be appropriate..
which may be done by simulating recoveries from different combination of
gross losses
Need to consider possibility of future management actions such as purchase of
further reinsurance after large event
..to model these dynamically would require much more complex methodology
Any other reasonable valid point

Part (i) – A large number of candidates did not outline the approaches as requested, just
giving the advantages and disadvantage. Most people described (a) well and most got a few
valid advantages and disadvantages. Too few of the points was the most common reason for
not getting marks. (b) was answered adequately, although even when “outlined” they often
did not specify what the ratios were applied to or just said applied to gross claims. Too few
candidates (if any) seemed to realise that net-to-gross ratios could vary by year and could be
used after diligent investigation.

Part (ii) – This was not well answered. Almost no-one actually referred to the table given in
the question or pointed out that different ratios would be used depending on underwriting
year taking into account the different stages of development they had reached, despite the
fact that we do not introduce red herrings in questions so they should have realised that the
table had some purpose. Too many of the candidates suggested that the ratios be applied to
ultimate claims rather than IBNR and quite a few seemed to think that they needed to
calculate net paid and incurred. There seemed to be little appreciation that this is actually a
sensitive method of netting down gross reserves that can allow for a great many things.

Part (iii) – Surprising poorly answered: it should have been a free mark for everyone. A lot
of candidates thought it was something to do with commission or an error or made
suggestions that made no sense at all.

Part (iv) – Poorly answered. Most got the 4% commission but many seemed to think that the
insurer would pay it to the reinsurer and nobody at all remembered that only 25% would be

Page 14
Subject ST7 (General Insurance: Reserving and Practical Modelling Specialist Technical) –
September 2012 – Marking Schedule

received because the treaty was only 25% ceded. A fair number of candidates gave answers
that were plausible alternative interpretations but did not explain enough to demonstrate that
they were not simply on the wrong track.

Part (v) – This was poorly answered This was left fairly open in the marking so that
candidates who demonstrated that they understood reinstatements receive marks but in
general they did not demonstrate any understanding.

Part (vi) – Not very well answered. Most candidates seemed to understood why it was
probably a poor procedure but few were particularly clear and only a very small number
thought about the possibility of reinsurance exhaustion at the upper end. Very few said that
as a first stab it was probably reasonable. Very few made reasonable attempts at suggesting
an alternative approach. Some suggested netting down the gross distribution so the mean is
equal to the mean of the net distribution which is essentially the same as applying net/gross
ratios with the same disadvantages. A large number explained how to get from a gross
reserve to a net reserve or suggested individual projections of large losses without explaining
how this gave a range.

8 (i)

Diagnostics

Development factors:

Uw Yr 1–2 2–3 3–4 4–5

2007 1.4753 1.1956 1.0835 1.0021


2008 1.5010 1.2013 1.0765
2009 1.5305 1.1898
2010 1.5573

Development factors 1–2/ percent of ultimate for dev year 1 for 2007 to 2010
Suggest that there is a trend in the 1–2 development
All other figures look reasonable: no trend over u/wtg years,
percent of ultimate/ development factors increasing in reasonable fashion,
year 3 and 4 % of ult/later development factors suggest may not be a tail or a very
small tail e.g. up to 1.0021
Adjust method by adjusting 2011 for trend
e.g. average difference 2008/2007; 2009/2008;2010/2009 is 0.0273
added to 2010 1–2 ratio gives 1.5847
Other reasonable ratios with satisfactory explanation

Page 15
Subject ST7 (General Insurance: Reserving and Practical Modelling Specialist Technical) –
September 2012 – Examiners' Report

Development Factors:

Year-on-Year 1.5847 1.1955 1.0799 1.0021 1.0000


Cumulative 2.0502 1.2938 1.0822 1.0021 1.0000

Future
Uw Yr Ultimate
Claims

2007 6,125 0
2008 6,624 14
2009 6,690 508
2010 7,501 1,703
2011 7,797 3,994
6,219

(ii) Interpolation formulae, assuming linear:

As at
Diag Uw Yr Interpolation Formula
31/12

2010 4 All 1/5 × 31/12/09 + 4/5 × 31/03/11


2008 2 2007 5/18 × 30/11/07 + 13/18 × 31/05/09
2008 1 2008 12/17 × 31/05/09
2007 1 2007 17/18 × 30/11/07 + 1/18 × 31/05/09

Restated Annualised Triangle of Claims Paid:

1 2 3 4 5

2007 1,841 2,848 3,595 3,868 3,940


2008 1,766 3,187 3,749 4,201
2009 2,405 3,730 4,578
2010 2,431 3,865
2011 2,810

Development Factors:

Devfs 1.6142 1.2209 1.0988 1.0187 1.0000


Cum 2.2059 1.3665 1.1193 1.0187 1.0000

Uw Yr Future
Ultimate
Claims

2007 3,940 0
2008 4,279 78
2009 5,124 546
2010 5,282 1,417
2011 6,199 3,389
5,430

Page 16
Subject ST7 (General Insurance: Reserving and Practical Modelling Specialist Technical) –
September 2012 – Marking Schedule

Diagnostics

Development factors:

Uw Yr 1–2 2–3 3–4 4–5

2007 1.5468 1.2625 1.0759 1.0187


2008 1.8045 1.1762 1.1207
2009 1.5509 1.2274
2010 1.5897

Linear interpolation (and extrapolation) used


… as only reasonable choice given the data supplied
If non-linear interpolation is used by a candidate then marks should be given
if assumptions are given and are reasonable
Diagnostics OK except:
4–5 development factor of 1.0187 suggests there may be a tail but 9 month
ratio from 31/03/11 to 31/12/11 of 3,940/3,936 or 1.0010 contradicts this and
suggests no tail is reasonable (or very small tail)
Although this also might suggest that linear interpolation is not reasonable and
that some other method of interpolation be used the simplest being graphical
And also 2008 dev year 1 not good: interpolation factor (here extrapolation)
strongly suggests linear interpolation not applicable
Recalculate with sum/sum ignoring 2008 dev yr 1

Development Factors:

Year-on-Year 1.5639 1.2209 1.0988 1.0187 1.0000


Cumulative 2.1371 1.3665 1.1193 1.0187 1.0000

Year Ultimate Future

2007 3,940 0
2008 4,279 78
2009 5,124 546
2010 5,282 1,417
2011 6,005 3,195
5,236

Part (i) – Generally reasonably well answered. A small number went on entirely the wrong
track discussing everything under the sun that they might do to reserve this business
sometimes without doing any calculations despite the question asking for a recalculation.
Those who calculated RTRs normally got most of the points required. Generally marks were
not lost if a tail factor higher than that suggested in the model answer was used although
marks were lost if RTR factors were calculated to only 2 decimal places.

Part (ii) – Many candidates constructed a standardised triangle of claims data from the data
given and calculated future claims more or less correctly but almost no-one looked at RTRs
as they did in part (i), and consequently did not adjust the development which they were

Page 17
Subject ST7 (General Insurance: Reserving and Practical Modelling Specialist Technical) –
September 2012 – Examiners' Report

directed to by the question stating “making any adjustments that you consider necessary”
and the fact that part (ii) carried 6 more marks than part (i).

END OF EXAMINERS' REPORT

Page 18
INSTITUTE AND FACULTY OF ACTUARIES

EXAMINATION

18 April 2013 (am)

Subject ST7 – General Insurance:


Reserving and Capital Modelling
Specialist Technical

Time allowed: Three hours

INSTRUCTIONS TO THE CANDIDATE

1. Enter all the candidate and examination details as requested on the front of your answer
booklet.

2. You have 15 minutes at the start of the examination in which to read the questions.
You are strongly encouraged to use this time for reading only, but notes may be made.
You then have three hours to complete the paper.

3. You must not start writing your answers in the booklet until instructed to do so by the
supervisor.

4. Mark allocations are shown in brackets.

5. Attempt all eight questions, beginning your answer to each question on a separate
sheet.

6. Candidates should show calculations where this is appropriate.

AT THE END OF THE EXAMINATION

Hand in BOTH your answer booklet, with any additional sheets firmly attached, and this
question paper.

In addition to this paper you should have available the 2002 edition of the Formulae
and Tables and your own electronic calculator from the approved list.

ST7 A2013 © Institute and Faculty of Actuaries


1 A general insurance company writes personal lines home insurance business. As the
result of a review of its claims experience the company concluded that, for business
written at the current premium rate, the combined ratio was 111%. It therefore
increased all its rates by 20%, effective for all business written from 1 September, to
target a 92.5% expected combined ratio. In the 3rd quarter £150m was written at the
old rates and £50m at the new rates. The premiums written each quarter were:

Q1 £200m
Q2 £220m
Q3 £200m
Q4 £200m

(i) Calculate the UPR and estimate the AURR as at the end of the year (ignoring
DAC), stating any assumptions made. [8]
(ii) Give reasons why the 92.5% expected combined ratio may not be appropriate.
[4]
[Total 12]

2 A general insurance company with a stable portfolio of business has had moderate
growth in recent years. The accounts for the financial year ending 31 December 2013
are forecast to be as follows:
£’000s
Written Premium 4,000
Underwriting Profit 50
Investment Income on Technical Reserves 10
Insurance Profit 60
Free Reserves at 31 December 2013 1,000

The company invests its free reserves in short-term deposits which yield 2.5% per
annum on the average of the free reserves at the beginning and end of the financial
year. The company pays no dividends or taxes. The company is considering a policy
of significant expansion and wants to know how this might affect its financial
position. It is assuming that the present rate of insurance profit, as a percentage of
written premium, and of investment return, will continue in each future financial year.

(i) Calculate the projected return on average capital employed for the year ending
31 December 2013. [3]
(ii) Calculate the maximum rate of growth that the company can sustain without
weakening its solvency margin (as a percentage of written premium). [1]

From 1 January 2014 the company is planning to increase its written premium at the
rate of 25% per annum for the next few years.

(iii) Project the company’s solvency ratio over the period to end 2015. [3]
(iv) Comment on the expected solvency ratios of the company over the period to
end 2015 [1]
(v) Calculate the ultimate solvency ratio of the company if the business continues
to expand at this rate for many years. [2]
[Total 10]

ST7 A2013–2
3 (i) Describe VaR in the context of general insurance. [2]

A general insurance company is diversified by writing property business in a large


number of different territories with around 25% of premium in each of the UK,
California and Japan. Because the company considers its business to be well
diversified, the only reinsurance purchased is quota share. The company’s current net
written premium income is £100 million. Over the past 10 years the combined ratio
has been as follows:

Year Combined Ratio


(x)

1 85%
2 92%
3 87%
4 88%
5 93%
6 89%
7 94%
8 95%
9 101%
10 98%

It has been suggested that the theoretical capital the company requires at the 99.5%
confidence level is as follows:

∑x ∑ ( x − μ)
2
μ= = 92.20% σ= = 4.79%
10 10

Capital = 2.807 × 0.0479 × £100m = £13.4 million

(ii) Explain why this answer is likely to be misleading. [7]


[Total 9]

4 (i) Explain how an insurer may generate a reserve range by bootstrapping the
Over-Dispersed Poisson (ODP) model. [6]

The company wishes to use the output in its capital model.

(ii) Outline the methods the insurer could use to allow for diversification between
reserving and underwriting risk. [4]
[Total 10]

ST7 A2013–3 PLEASE TURN OVER


5 As part of a new regulatory framework, a motor insurer is required for the first time to
estimate reserves on a discounted basis.

(i) Outline the issues to be considered when determining appropriate discount


rates. [3]

The following information has been provided for a particular sub-class of business:

Held reserves as at 31/12/2012

Underwriting Outstanding IBNR (£m)


year reported claims (£m)

2009 85.0 44.5


2010 145.0 70.5
2011 150.0 156.0
2012 80.0 322.5

Actuarial development profiles

Annual period 0–1 1–2 2–3 3–4 4–5 5–6 6–7

Paid link ratios 5.000 2.165 2.825 1.825 1.285 1.095 1.000
Incurred link ratios 6.000 2.569 1.432 1.210 1.095 1.015 1.000

The company has agreed to use a discount rate of 4% per annum.

(ii) Show that the discounted value for the reserves as at 31/12/2012 is £957.9m,
stating any necessary assumptions. [7]

(iii) Derive the discounted mean term to settlement for the 31/12/2012 reserves [2]

(iv) Suggest an investment strategy that may be an appropriate match for the
insurer’s liabilities. [3]
[Total 15]

6 (i) List the principal types of liability insurance. [2]

(ii) Describe the claim characteristics of liability insurance. [4]

In a particular jurisdiction, a type of liability insurance has traditionally been


written on a losses occurring basis. Insurers now intend to change to a claims-
made basis.

(iii) Discuss the problems that this transition may cause. [8]
[Total 14]

ST7 A2013–4
7 A medium-sized general insurance company that writes personal and commercial
lines property and liability business has recently appointed a new claims manager. He
has introduced new procedures and a new instruction manual for setting claims
reserves on outstanding claims. These changes were introduced about three months
before the end of the financial year. All case reserves that have been set since then
have used the new procedures. Other case reserves are being revised in line with the
new procedures and this process is expected to take about a year.

(i) Discuss the potential effects of this change on the actuarial reserving process
at the end of the year and the likely effect on the results of the calculations. [3]

(ii) Suggest the steps that may be taken to keep the process as accurate as
possible, both in the first and subsequent years, commenting on their potential
effectiveness. [12]
[Total 15]

8 A small general insurance company writing property and liability business, although
solvent, has recently stopped underwriting and entered run-off.

Discuss appropriate considerations that it should make in setting its investment policy,
suggesting appropriate investment types. [15]

END OF PAPER

ST7 A2013–5
INSTITUTE AND FACULTY OF ACTUARIES

EXAMINERS’ REPORT
April 2013 examinations

Subject ST7 – General Insurance:


Reserving and Capital Modelling
Specialist Technical

Introduction

The Examiners’ Report is written by the Principal Examiner with the aim of helping
candidates, both those who are sitting the examination for the first time and using past papers
as a revision aid and also those who have previously failed the subject.

The Examiners are charged by Council with examining the published syllabus. The
Examiners have access to the Core Reading, which is designed to interpret the syllabus, and
will generally base questions around it but are not required to examine the content of Core
Reading specifically or exclusively.

For numerical questions the Examiners’ preferred approach to the solution is reproduced in
this report; other valid approaches are given appropriate credit. For essay-style questions,
particularly the open-ended questions in the later subjects, the report may contain more points
than the Examiners will expect from a solution that scores full marks.

The report is written based on the legislative and regulatory context pertaining to the date that
the examination was set. Candidates should take into account the possibility that
circumstances may have changed if using these reports for revision.

D C Bowie
Chairman of the Board of Examiners

July 2013

© Institute and Faculty of Actuaries


Subject ST7 (General Insurance: Reserving and Practical Modelling Specialist Technical) –
April 2013 – Examiners' Report

General comments on Subject ST7

Candidates who are well prepared generally appear to perform reasonably on ST7, with the
more challenging questions tending to occur on SA3. Candidates should consider the
following advice however (if they are not already):

• Lists are hugely valuable for breadth of point generation but candidates should always
exercise judgement when applying them.

• Calculation questions will come up on a regular basis with ST7, as candidates can clearly
observe from examination of historical papers. Candidates should always be prepared for
such staples as balance sheet preparation, triangle manipulations and projections and
reinsurance layer calculations (along with being able to carry out any necessary
adjustments including inflation, exposure and time period issues).

• Capital questions should be expected on every paper and represent a sufficient proportion
of the course content that candidates should not expect to be able to pass on their
reserving knowledge alone. Those who do not encounter capital work in their
professional lives should be particularly careful to ensure that they take time to
familiarise themselves with this element of the course.

• Candidates should aim to be able to give near exact glossary definitions as incoherent or
vague descriptions will be marked harshly. If candidates struggle to remember
definitions verbatim they should take the time to properly analyse the glossary definition
to ensure they have fully absorbed all the nuances of the definition.

• It is important to always read the question properly.

Comments on the April 2013 paper

See comments under individual questions.

Page 2
Subject ST7 (General Insurance: Reserving and Practical Modelling Specialist Technical) –
April 2013 – Examiners' Report

1 (i) Assumptions:
Policies annual
Risks earned evenly over each policy year
Policies commence mid-period on average
Expenses remain the same after the premium rate increase
Mix of business remains the same after the premium rate increase/for each
period
The calculation of UPR and URR is on an annual basis to calculate the AURR
for the year (i.e. credit is taken for periods when URR < UPR)
Everything is gross of reinsurance or no details about reinsurance have been
given so no adjustments are possible in the calculations to allow for
reinsurance
Combined ratios accurate

Calculations:
Combined ratio at the new rate = 111/120 = 92.5%

Period Written Unearned UPR Combined URR AURR


Premium Exposure Ratio

Q1 200 1/8 25.00 111.00% 27.75 2.75


Q2 220 3/8 82.50 111.00% 91.58 9.08
Q3 @ old rate 150 7/12 87.50 111.00% 97.13 9.63
Q3 @ new rate 50 17/24 35.42 92.50% 32.76 -2.66
Q4 200 7/8 175.00 92.50% 161.88 -13.13
Total 405.42 411.09 5.67

AURR = 411.09 – 405.42 = 5.67

A fairly straightforward calculation which very few got right. The main errors were not
splitting the calculations for the third quarter i.e. assuming premium received half-way
through the quarter rather than at the respective mid-points of each period and/or not
assuming AURR is on an annual basis and therefore taking AURR at new rates as zero.
Some calculated UPR with not splitting 3rd quarter and URR with splitting 3rd quarter which
is not logical. Some candidates wrongly made adjustments to the UPRs for the rate changes.

(ii) The starting point assumption of 111% combined ratio is itself an estimate and
may be incorrect
The mix of business may change: bad risks may stay and good risks may leave

The decrease in combined ratio will be smaller in such circumstances.

Competitors may also have increased their rates.


The impact on the mix of business and hence on combined ratio will be
affected by the size of their increases.

Management expenses may not be as assumed


The decrease in combined ratio may therefore be lower than forecast .

Page 3
Subject ST7 (General Insurance: Reserving and Practical Modelling Specialist Technical) –
April 2013 – Examiners' Report

If rate increases lead to a fall in premium volumes this may not be


accompanied by a fall in management expenses
If volumes are maintained then the 20% increase will raise the denominator so
the expense ratio would be lower

To keep the business volume at the earlier level acquisition expenses may
need to be increased.
This will increase the combined ratio to above the target.
Any answer based on actual losses being different from expected e.g.
“unexpected large losses” is not a valid answer as the question refers to
expected loss ratio, not its outcome

Other external factors such as:


Claim inflation assumptions being wrong
.. or if the rates are before claim inflation then this should be taken account of
in the loss ratio expectation

Reinsurance: any reasonable point e.g. If RI premium was fixed, impact of


premium increase on net combined ratio could be greater than expected
Any other reasonable generic points

The most common error here was not considering the combined ratio as loss ratio plus
expense ratio and hence not commenting on expense effects. Some answered the question of
why the company should not be targeting a 92.5% ratio than rather than the question asked.

2 (i) Let the free reserves at 1.1.13 be A.


SM at 31.12.13 = SM at 1.1.13 + insurance profit + yield on SM
1000 = A + 60 + 0.025 × (A + 1000)/2
Giving A = 916
Average capital employed = (916 + 1000)/2= 958
Yield on capital employed = 0.025 × 958 = 24
Projected return on capital = (60+24)/958 = 8.8%

Very few candidates got anywhere with this question with hardly any attempts at any algebra.
A very similar question has been asked before (in 1989) with less problems. For part (i) a
large number of candidates assumed that free reserves b/fwd were the same as those c/fwd
which is not possible as the question states there has been moderate growth. Many others
took the free reserves b/fwd as those c/fwd less insurance profit thus ignoring investment
income on the free reserves and the investment income on free reserves was often not
included in the return on capital calculation. In this and other cases where the two free
reserve figures were different candidates often calculated the return on capital employed at
the year-end rather than on the average capital as requested in the question.

Page 4
Subject ST7 (General Insurance: Reserving and Practical Modelling Specialist Technical) –
April 2013 – Examiners' Report

(ii) Let the required level of growth be g.


SM at 31.12.14 = SM at 1.1.14+ insurance profit + yield on SM
1000(1 + g) = 1000 + 60(1 + g) + 0.025(1000)(1 + g/2)
Giving g = 9.2%

There were almost no reasonable answers to this part. Favourite answers illogically were
given as 2.5% being the yield on free reserves and 25% being free reserves c/fwd divided by
written premium.

(iii) The projection is as follows, using equivalent formulae to those above:

2013 2014 2015

Premium 4000.0 5000.0 6250.0


Insurance Profit 60.0 75.0 93.8
Yield on SM 24.0 26.3 29.1
Total Return 84.0 101.3 122.8
SM at end of year 1000.0 1101.3 1224.1
SM% 25.0% 22.0% 19.6%

Marks were available for calculating the premium and insurance profit and making an
attempt at the rest but many did not do even this. Some calculated using the formula free
reserve c/f = free reserve b/f + insurance profit + (free reserve b/f + ½ insurance profit) x
2.5% which while not being quite correct is a reasonable approximation giving SM at end
2014 of 1100.9 and end 2015 of 1223.4 and correct SM%’s at 1 d.p.

(iv) It can be seen that the solvency ratio of the company will reduce over the
period.
This is because in any given year the total return on the solvency margin is
less than that necessary to increase the initial solvency margin by 25%.
Because the growth in yield lags the growth in premium the solvency margin
decreases over time

Many candidates did state that solvency margins would be reducing and gave a reasonable
explanation thereby getting the mark for this part.

(v) If growth continues at this rate then the solvency will cease to fall once it has
reached a certain level (m%).
Using the standard formula, with premium P:
mP = mP/1.25 + 60/4000.P + 0.025(mP/1.25 + mP)/2
Giving m = 8.5%

Almost nobody made a reasonable attempt at this part. A large number of candidates stated
that the ultimate solvency ratio would reduce to zero which a bit of thought would have told
them was not logical.

Page 5
Subject ST7 (General Insurance: Reserving and Practical Modelling Specialist Technical) –
April 2013 – Examiners' Report

3 (i) Value at Risk


The loss at a predefined confidence level (e.g. 99.5%).
…over a fixed time period e.g. a year
Consequently if an insurer holds capital at the VaR it will remain solvent with
probability of the confidence level (e.g. 99.5%)
and be insolvent with probability of one minus the confidence level (e.g.
0.5%)

Reasonably well answered on what VaR is although some candidates thought this it was a
confidence interval or probability rather than a loss at a given probability. Very few
candidates stated that it is over a time period. Not many candidates described how it is used
in general insurance.

(ii) In formula for s.d. should be 9 for sample rather than 10 for whole population

There is a clear (upwards) trend in the loss ratios


..so that formula for s.d. needs to be adjusted

If there is an insurance cycle it is obscured by this trend (or, unlikely, there is


an upwards trend of around 10 years as part of the cycle which is rather long)

More importantly, half of the company’s business is in territories subject to


earthquake and other catastrophic risk
… and the company has apparently been fortunate in not being hit by a
catastrophic event in the 10 year period
… however, the probability of a catastrophic event (100-year?) has to be
allowed for in the calculation of the VaR

The 2.807 multiplier is the correct confidence factor at 99.5% confidence level
…but for a 2-sided distribution: we are only interested in the downside risk so
a 1-sided distribution would be more appropriate
…for which multiplier is 2.576 for 99.5%
.. . but the assumption of a normal distribution is invalid as the distribution is
positively skewed
… and a much larger multiplier is required

Other factors to consider:


The capital required should not just be related to the confidence level but
should be offset e.g. by expected profit and investment return
..and regulatory capital could well be different to theoretical capital
A straight average may not be appropriate e.g. could be weighted by volume
of business
There may be a better measure of exposure than the premium used e.g.
exposed to risk or sum insured
Overall business level may not be appropriate for the calculation: possibly
should sub-divide e.g. by territory
Mix of business may be changing
Volume of business may be changing
Terms and conditions/ management expenses etc. may be changing

Page 6
Subject ST7 (General Insurance: Reserving and Practical Modelling Specialist Technical) –
April 2013 – Examiners' Report

Combined ratio is usually claims incurred/earned premium + expenses/written


premiums giving complications
The combined ratios must be estimates, particularly for more recent years, and
therefore may be inaccurate.
Changes in reinsurance arrangements
The above is the underwriting risk: other risks should also be taken into
account
Other risk examples
Any other generic points

Many candidates failed to relate their answers to the details given in the question, such as the
increasing combined ratios, the relative stability of the ratios, the writing in catastrophe-
exposed territories with only quota share reinsurance, the 2.807 standard deviations in the
formula and the 10 rather than 9 used in the formula for σ. A few candidates stated that the
combined ratios were very variable rather than the exact opposite applying in the
circumstances. Some candidates got side-tracked into spending too much time on other risks.

4 (i) Determine the nature of the range required – a distribution of possible


outcomes (rather than best estimates) is most appropriate for an ODP model
Collect data required namely claims triangles, selected development patterns
and booked reserves.
Can be paid or incurred claims data
..but may be problems with incurred data because may be negative increments

Data can be gross or net of reinsurance


.. but could be problems with net e.g. as may be distorted or may have
negative increments
Segment data into homogeneous groups if not already done so
.. or combine if data in some reserve classes are too sparse for stochastic
modelling.
Fit the distribution based on the incremental claims triangles
..so that the mean is equal to the booked reserve for each class of business
ODP distribution assumes variance is proportional to
..and greater than the mean.
Obtain the distribution around the ODP model by using the bootstrapping
technique
..which involves calculating expected values and residuals for each point in
the claims triangle
..resampling (with replacement)
..from residuals to obtain new triangle (pseudo-data)
..refitting the chain ladder model to the new triangle to obtain new reserve
estimate
Repeat a large number of times
Correlations across multiple lines of business should be considered
..using methods such as copulas..
..in order to derive a range for the reserves at a total level.
Summarise results by recording mean/variance/percentiles
Any other generic points

Page 7
Subject ST7 (General Insurance: Reserving and Practical Modelling Specialist Technical) –
April 2013 – Examiners' Report

Most candidates got the basics of describing the bootstrap but little else e.g. the data
(paid/incurred, gross or net, negative increments etc.)

(ii) Assumptions used in underwriting and reserving risk can be linked


..such as common future claims inflation/ future loss ratios linked to past.
Explicit correlation factors between distributions...
..such as defining a correlation matrix that links underwriting and reserving
risk.
Correlations could be blocks or over years
Correlations could be by class between underwriting and reserving risks with
higher correlation for non-catastrophe classes
Copulas could be best approach as dependency between risks likely to be
complex
..e.g. Gumbel copula that allows for stronger dependency in tail.
Implicit correlations may be in place between different scenarios in model
..such as future tort reform worsening both reserves and future business or
other example
Deterministic allowance for diversification unlikely given that stochastic
model used

Those who knew the bookwork got some quick marks here although many did not answer the
question set e.g. talking rather about allocating capital.

5 (i) New regulatory regime should be considered


.. it may prescribe particular rates, or means of calculation, that should be
used
Purpose of the exercise – best estimate or more prudent discounting basis
Consistency with return on assets held by insurer
..including if business written overseas where rates may be materially different

Allowance for level of tax/investment expenses incurred by company


..and any non investible assets held by insurer.
Rates that are used by competitors for consistency
Term and nature of the liabilities by claim type
Any generic point related to motor

Generally reasonably answered although hardly anyone made reference to this being a new
regulatory framework.

(ii) Fully developed on a paid basis after 6 years (no tail required)
Yield curve is appropriate match by currency for the liabilities
Use paid development profile
Convert to percentages developed
Re-base profile for each underwriting year
Calculate assuming payments made half way through year

Page 8
Subject ST7 (General Insurance: Reserving and Practical Modelling Specialist Technical) –
April 2013 – Examiners' Report

Calculation:

Development Yr 0–1 1–2 2–3 3–4 4–5 5–6 6–7

Paid link ratios 5.000 2.165 2.825 1.825 1.285 1.095 1.000

Cumulative %
developed 1.3% 6.4% 13.8% 38.9% 71.1% 91.3% 100.0%

Incremental %
developed 1.3% 5.1% 7.4% 25.2% 32.1% 20.3% 8.7%

Outstanding at
start of year 98.7% 93.6% 86.2% 61.1% 28.9% 8.7%

2009 uw yr: 129.5 × (32.1 × 1.04−½ + 20.3 × 1.04−1½ + 8.7 × 1.04−2½) / 61.1 = 124.0

2010 uw yr: 215.5 × (25.2 × 1.04−½ + 32.1 × 1.04−1½ + 20.3 × 1.04−2½ + 8.7 × 1.04−3½) / 86.2
= 202.2

2011 uw yr: 306.0 × (7.4 × 1.04−½ + 25.2 × 1.04−1½ + 32.1 × 1.04−2½ + 20.3 × 1.04−3½ + 8.7
× 1.04−4½) / 93.6 = 277.9

2012 uw yr: 402.5 × (5.1 × 1.04−½ + 7.4 × 1.04−1½ + 25.2 × 1.04−2½ + 32.1 × 1.04−3½ + 20.3
× 1.04−4½ + 8.7 × 1.04−5½) / 98.7 = 353.8

Total discounted reserves = 124.0 + 202.2 + 277.9 + 353.8 = 957.9

This is a fairly straightforward calculation but very few got it right and those that did
produce correct (or nearly correct) solutions did tend to make heavy weather of it by making
it over-complicated. A fair number of candidates assumed that the ratios given were
cumulative while it should be clear they are not because the question refers to link ratios, the
headings are 0–1, 1–2,… a comparison of paid and incurred ratios shows not possible e.g.
paid/incurred 5.000 and 6.000 for 0–1 and 2.825 and 1.432 for 2–3 and also assuming
cumulative for the paid ratios gives strange paid link ratios including a large negative one.
A lot of incorrect methods were used such as using paid ratios for outstanding claims,
incurred ratios for IBNR and also somehow trying to get ultimates and then work back.
However, most of these methods did not get vary far before the candidate gave up and there
were a lot of no attempts at this part.

(iii) Undiscounted reserves = 1053.5m


Discounted Reserves = 957.9m
Discount Factor = Discounted Reserves/Undiscounted Reserves =
957.9/1053.5 = 0.909252
Discount Factor = (1 + i)−DMT
Taking logs of both sides
log (0.909252 )= −DMT × log(1.04)
DMT = 2.43 yrs (2 d.p)

Page 9
Subject ST7 (General Insurance: Reserving and Practical Modelling Specialist Technical) –
April 2013 – Examiners' Report

This was very straightforward using just the answer given in part (iii) and the total reserve
from the question but many did not attempt or tried to calculate using figures worked out
(usually wrongly) in part (ii). In some cases they came up with answers that were obviously
wrong without comment (e.g. greater than 7 years when the link ratios given are 1.000 for 6–
7).

(iv) Mixture of short term investments as match for damage claims


..with longer term investments as match for injury losses
If available, inflation related assets better match for liabilities..
..though claims inflation may be very different from published local indices
(e.g. RPI / CPI)
May use structured settlements such as PPOs and/or purchases annuities where
appropriate
Depending on currency of liabilities may need assets denominated in other
currencies.
Generally less volatile assets such as government bonds more appropriate for
matching liabilities
Only if sufficient free assets should assets such as equity and property be
considered

Those who did best were those who related the assets to the those required for damage claims
and those for injury claims rather than trying to match against the mean term from (iii).

6 (i) Employers’/workers’ compensation


Public
Products
D&O
E&O
Environmental
Motor third-party
Marine
Aviation
Professional

Most candidates got full marks for this bookwork.

(ii) Liability
Liability is insurance against the risk of being held legally liable to pay
compensation to a third part
Claims are typically for property damage or bodily injury
For a claim to be payable the policyholder must show that the insured is liable
Amounts
The claim amount distribution is typically positively skewed
Depends on class e.g. for motor there may be small claims for property
damage and large claims for bodily injury
Also, amounts may depend on the generosity or otherwise of arrangements for
compensation in particular jurisdictions.

Page 10
Subject ST7 (General Insurance: Reserving and Practical Modelling Specialist Technical) –
April 2013 – Examiners' Report

Delays
All of the following may take longer than for other classes e.g. for bodily
injury claims
A delay between the incident occurring and the policyholder being aware of it
A delay between the insured becoming aware of the loss and reporting it
A delay before sufficient details of the incident can be gathered to assess the
value of the claim
A delay until an injured party’s condition stabilises to the extent that
assessment of damages is appropriate
A delay in agreeing the actual value at which the claim is to be settled, and the
payment of this amount to the insured
The amount of claim may be affected by considering whether there is
contributory negligence
Latency
In particular some diseases may take more time to manifest themselves after
they have been caused:
the classic example is mesothelioma, which may not begin until some decades
after the causative exposure to asbestos.
Frequency
Depending on class there may be a large number of small claims and small
number of large claims
There can be accumulation of claims e.g. for professional liability
Inflation
Claims may be subject to higher than standard measures of inflation
Other
The treatment of all aspects of liability claims differs more between different
jurisdictions than is normal for other classes.
Claims may be contested in court. (This is true of all claims, but especially of
liability)
This increases delays and affects amounts, frequency and inflation

Reasonably well answered in general. Several needed to say more than they did on latent
claims such as what they are and the periods involved.

(iii) Occurrence policies provide cover for all the losses that the insured has caused
during the period of the policy.
Claims-made policies cover all those that were reported during the policy
period.
This means that when the market moves to claims-made many of the losses
that are reported will be covered under the old, occurrence, policies.
Claims that occurred under the old system will need to be excluded from the
new;
in classes and jurisdictions much prone to latent claims this need for exclusion
will last for a long time.
This means that early claims-made policies will have significant exclusions,
and will be subject to substantial discounts on premium that will need to be
withdrawn over time.
This means that the total income of the sector will be depressed,
with implications for expense loadings and commission rates.

Page 11
Subject ST7 (General Insurance: Reserving and Practical Modelling Specialist Technical) –
April 2013 – Examiners' Report

If the whole of the market does not change at once then policy exclusions will
need to be drawn up individually for different insureds,
with implications for underwriting procedures.
Claims-made policies will not provide cover for any losses reported after
expiry date.
This will create a need for run-off policies.
For example, professional people who retire may still be sued for their actions
while still in practice and will need this cover;
policies will need to be developed to meet this need.
There will also be problems with businesses that are closed, become bankrupt
or merge into others.
Because of these problems legal restrictions may be placed on the cover that
may be offered
that may restrict insurers’ ability to make this change fully.

Problems are created for pricing and reserving


in particular as the development period is much shorter for claims made than
for occurrence.
May be problems with outwards reinsurance e.g. ensuring no gaps
Policyholder communication required e.g. to ensure they know what their
coverage is
Policyholder protection e.g. to ensure that no lawsuits are brought in the future

This was in general poorly answered. A large number talked about possible gaps but they
were missing the point that the main problem is overlap. Some mentioned overlap but very
few realised that the way this problem is overcome is that for the new claims made policies
(the first one being called a first-year claims made policy etc.) would not cover losses already
covered by previous occurrence policies i.e. exclusions would apply/ there would be a
retroactive date (as defined in the core reading). Later on there may be gaps
e.g. professional persons will have to consider what insurance is available after they retire,
but again not many mentioned this even when they said gaps were a problem. A lot of
candidates talked about finding occurrence cover from another insurer but the question does
suggest that the whole market is now changing to claims made for the type of liability
insurance being considered which is logical as there being the two different bases in the
same market would be impractical. A particular real-life example is when the whole US
medical malpractice market changed from occurrence to claims made a few years ago. One
minor point: defining claims insurance made as being for those claims made in the policy
period is a tautology not a definition.

7 (i) The changes will affect the development patterns of incurred claims
that make up the data triangles that the reserving actuary is likely to use in
reserving.
Assuming that annual or quarterly triangles are used, only the last diagonal of
the incurred triangle will be affected:
it seems reasonable to assume that the payment of claims will not be affected
by the change.
If the actuary uses the incurred chain-ladder method or the incurred
Bornhuetter-Ferguson method to project ultimate claims then using the last

Page 12
Subject ST7 (General Insurance: Reserving and Practical Modelling Specialist Technical) –
April 2013 – Examiners' Report

diagonal will give inappropriate results since the values projected from are not
in keeping with the patterns of development inherent in the rest of the
triangles.

(ii) One approach to reserving in these circumstances is to use only methods based
on paid claims.
Or to put more weight on paid-claim development.
This should lead to appropriate results, but in some classes the development of
paid claims may be so slow that recent years’ results are not reliable;
this is likely to be especially true of the liability classes.
Also, if incurred methods have typically been used in the past but are now
abandoned then it may not be easy to assess the effects of this change.
Finally, it is undesirable wholly to ignore the information available in the
outstanding claims.
If the actuary wishes to continue to use the incurred claims, either for selecting
ultimate claims or for comparative purposes, then it is necessary to understand
the effects of the changes.
Should talk with the claims manager and other stakeholders
.. and compare the new instruction manual with previous instructions
.. to see if possible to do a what-if analysis.
Comparing the new policy to the old may give some indication of the extent
and direction of the likely changes to reserves as a result of the change in
policy
– these effects need not be straightforwardly positive or negative.
About a quarter of old claims are likely to have been amended by the valuation
date –
the actuary could analyse the extent of increases and reductions.
Such an analysis may enable him to adjust the last diagonal of the incurred
claims
so that it is consistent with the interior of the triangle.
If triangles of numbers of claims outstanding are available
then the actuary can calculate a triangle of average outstanding claim amounts.

If past reserving has been consistent


then all the values in each column of the triangle (with claims as single
development periods) should be roughly equal,
or at least affected only by inflation and random variation.
The extent to which the latest entry is out of line with the trend of those before
it may indicate the effect of the changes
and provide a basis for adjustment.
Granularity shoud be considered; sub-dividing the classes of data by type of
claim may allow greater insight
Other possible approaches include the use of benchmarks,
if the new policy is reasonably close to general market practice,
and projecting the ultimate claims from the penultimate diagonal.
The latter has the problem of ignoring actual development in the most-recent
year.
An inflation adjusted methodology may assist in identifying differences
caused by the new process

Page 13
Subject ST7 (General Insurance: Reserving and Practical Modelling Specialist Technical) –
April 2013 – Examiners' Report

Also, could adjust all past history to be on same basis as new process
This is initially more complex but has the advantage that for future valuations
the data is already there and becoming more relevant
This is the Berquist Sherman method of adjusting historical development for
changes in case reserving.
In the following financial year all outstanding claims will have been set under
the new policy.
However, as the previous diagonal was a mixture of policies it is the only one
that will have been.
It may still be adjusted, but as two diagonals will now need adjustment the
result of the reserving exercise may be very sensitive to the actuary’s
adjustments.
As the years go by there will be more diagonals on the new basis and it will be
appropriate to use them to estimate development factors.
Any operational point, e.g. peer review
The process should be monitored over time so that the methodology can be
adapted appropriately
Generic points on keeping the process of case reserving accurate

There was a lot of overlap between the answers given for parts (i) and (ii) of this question
which was allowed for when marking. This was a very poorly answered question. In part (ii)
candidates are asked to suggest steps to keep the process as accurate as possible, with the
process intended to be the actuarial reserving process. However, some candidates assumed
it meant the case reserve revision process. Some marks were made available for suggestions
on making the case reserving revision process accurate but the majority of candidates did
concentrate on the actuarial reserving process. In general attempts were poor with few
practical suggestions and insufficient detail for anywhere near 15 marks. Many candidates
failed to realise that incurred claims development would be affected but not paid claims and
that consequently methods based on paid claims should be considered thereby losing several
available marks. Also candidates often only considered the first year and not subsequent
years as requested,

8 The prime objective in the investment of assets is to maximise investment return,


subject to meeting all contractual obligations and recognising the uncertainty
involved.
This company, although solvent, is unlikely to have large amounts of free assets,
since otherwise it would be unlikely to have been closed.
This means that there is not likely to be a great deal of freedom to invest
adventurously,
The company will need to make a projection of its liability outgo to extinction
and ideally it ought to invest in assets that match it reasonably closely.
This matching will need to be done by term and by currency,
although it is possible that a small company may have assets and liabilities all in a
single currency.
However, the company has only recently closed to business; for some months at least
it may have exposure to property catastrophes,
for which a degree of liquidity is required.
There will be no premium income going forward so a greater need for liquidity.

Page 14
Subject ST7 (General Insurance: Reserving and Practical Modelling Specialist Technical) –
April 2013 – Examiners' Report

..although as has just gone into run-off there will be some premium income in the
short term
There may be some regulatory constraints on a company in run-off which are likely to
be more stringent than for an on-going company
The constraints will likely be exacerbated as this is a small company
Expenses will be more significant in run-off including redundancy costs increasing
the need for liquidity.
There may be a need for more cash to be held for liquidity
..although possibly reduced as in run-off less expectation of spikes from catastrophes
The amount held in non-investible assets should be considered (any examples)
Also any credit facilities being used should be considered as will reduce need for
liquidity
In addition, a company in this situation is usually looking for opportunities to
commute its liabilities favourably,
and to take advantage of any such opportunities it may need to be able to liquidate
assets quickly.
The desirability of liquidity suggests that it may be appropriate to invest at a shorter
term than matching would imply.
The company’s liabilities are almost all real in nature/affected by inflation,
even if all the payments are affected by a number of different inflation rates.
The liability claims may take more than ten years to settle, and possibly much longer
if there are likely to be latent claims.
This argues for a hedge against inflation in its investment selection.
Depending on the local valuation rules there may be a need to avoid excessive
volatility.
For example, if reserves are valued on an undiscounted basis and assets at market
value and if interest rates rise then there is a possibility that the company might
become insolvent even if its projected asset income fully covered its liability outgo at
all times.
The need to avoid volatility in the value of assets means that the company will need to
avoid investing in equities.
It will also be impossible to match the term of assets to liabilities using equities.
This is despite the fact that equities provide a hedge, albeit an imperfect one, against
inflation.
If the liability account does include some extremely long-tailed claims, such as latent
disease, the need for a long-term hedge might outweigh these considerations.
The desire to maximise investment income argues for the inclusion of some high-
grade corporate bonds in the portfolio.
The need for liquidity argues for the use of government securities to match the
projected outgo.
It also argues for the use of earlier maturities than would match the projected outgo.
The need to remain solvent on particular accounting rules may also argue for
investing shorter than the term of the expected liability outgo.
Index-linked stocks may suit all purposes, including hedging against inflation,
but they are not available in all jurisdictions and often only for government bonds,
which is likely to reduce yield.
The price of these may also be prohibitive
particularly if pushed up by pension-fund investors.
The assets currently held are important.

Page 15
Subject ST7 (General Insurance: Reserving and Practical Modelling Specialist Technical) –
April 2013 – Examiners' Report

If the company has substantial equity holdings as currently matched assuming future
new business and/or because the company was growing
then a disinvestment strategy will need to be considered.
Timing will need to be considered as to how long it should take to get to a desirable
portfolio.
It may need to consider trigger points for when to disinvest from equities.
Outwards reinsurance is important
…this will affect the volatility with stop-loss if available being particularly effective
in reducing volatility and the need for liquidity
Tax will need to be considered.
E.g. as just gone into run-off likely to have tax losses brought forward
In summary:
the portfolio is likely to include high-quality corporate bonds
and government stock,
invested slightly shorter than the expected liability outgo,
with a proportion of index-linked stock if available.
Equities are unlikely to be an appropriate part of the portfolio, although it is not
impossible.

Generally reasonably well answered. The main failure was in not relating the answer to the
specifics given in the question, e.g. suggesting using equity and sometimes property
investment which are of course inappropriate as it is both a small company and in run-off.
Also many did discuss the considerations to be made but were unclear in suggesting
appropriate investment types. Some candidates stated that expenses would be reducing
thereby missing the point that expenses would be more significant in run-off. Also, what
competitors are doing is unlikely to be relevant as in run-off the company is not competing.

END OF EXAMINERS’ REPORT

Page 16
INSTITUTE AND FACULTY OF ACTUARIES

EXAMINATION

26 September 2013 (am)

Subject ST7 – General Insurance:


Reserving and Capital Modelling
Specialist Technical

Time allowed: Three hours

INSTRUCTIONS TO THE CANDIDATE

1. Enter all the candidate and examination details as requested on the front of your answer
booklet.

2. You have 15 minutes at the start of the examination in which to read the questions.
You are strongly encouraged to use this time for reading only, but notes may be made.
You then have three hours to complete the paper.

3. You must not start writing your answers in the booklet until instructed to do so by the
supervisor.

4. Mark allocations are shown in brackets.

5. Attempt all eight questions, beginning your answer to each question on a separate
sheet.

6. Candidates should show calculations where this is appropriate.

AT THE END OF THE EXAMINATION

Hand in BOTH your answer booklet, with any additional sheets firmly attached, and this
question paper.

In addition to this paper you should have available the 2002 edition of the Formulae
and Tables and your own electronic calculator from the approved list.

ST7 S2013 © Institute and Faculty of Actuaries


1 Define fidelity guarantee insurance and directors’ and officers’ liability insurance,
identifying the key difference between them. [2]

2 A general insurance company writing product liability insurance is considering its


investment options. The Government has recently issued a new index-linked
government loan which pays a nominal coupon of 1% per year. Both the coupon and
the capital value of the loan are indexed to the local consumer price index. It matures
in 10 years’ time.

Explain whether the stock should be purchased by the company. [8]

3 A large multinational insurer has introduced asset/liability modelling (ALM) across


its businesses in different countries.

(i) List the variables that are likely to be modelled within the economic scenario
generator section of the model. [2]

(ii) Suggest interdependencies between assets and liabilities that should be


considered for representation within the ALM framework. [4]

(iii) Outline key operational controls that should be in place around the
assumptions that are used in the model. [4]
[Total 10]

4 An insurance company intends to introduce a new product that would protect


homeowners against future negative equity (a shortfall in market value relative to
outstanding loan amount) when their houses are sold. The product would involve a
proportion of the house cost being retained by the insurer on purchase of the property.
The insurer would then indemnify the policyholder against any shortfall on the sale of
the property subject to independent valuation.

(i) Comment on the appropriateness of possible distribution channels for the


product. [3]

(ii) Describe the specific risks for the insurer from launching this particular
product. [7]
[Total 10]

5 In projecting actuarial estimates, three main types of uncertainty can be distinguished.

Describe each type of uncertainty, using examples to illustrate the differences between
them. [6]

ST7 S2013–2
6 The management of a general insurance company has been concerned that for some
years its claims handling processes have been ineffective. Delays have become
common, claims have been paid that should have been rejected and excessive amounts
have been paid on valid claims.

A new claims-handling protocol has been developed to remedy these problems and
there has been extensive staff training. The new procedures are to be introduced on
1 January 2014. The senior management of the company have discussed the project
with the reserving actuary and it is evident that they expect the reserves as at
31 December 2013 to reflect the prospective improvements.

Discuss the issues that the actuary should consider and the extent to which he should
allow for them when taking account of these changes in his work on the year-end
reserves. [12]

7 (i) List the advantages and disadvantages of an insurer writing business through a
Lloyd’s syndicate. [4]

The table below shows the initial summary results for the capital model of a Lloyd’s
syndicate.

Risk type Capital requirement (£m)

Reserving risk 405


Underwriting risk 120
Aggregate insurance risk 550
Market risk 35
Counterparty default risk 85
Operational risk 15
Liquidity risk 25
Total syndicate capital requirement 475

(ii) Comment on the amounts shown in the table, considering both the
reasonableness of the figures and what they show about the syndicate. [11]

The capital model outputs have been reviewed by Lloyd’s, and the managing agency
has been advised that the syndicate would require a higher level of capital to support
its business plan for the next underwriting year.

(iii) Describe options available to the syndicate. [6]


[Total 21]

ST7 S2013–3 PLEASE TURN OVER


8 A small general insurance company writes liability insurance through regional brokers
and the London Market. The company is newly established, having started writing
risks on 1 July 2012. The professional indemnity class of business currently accounts
for most of the business written by the company. The company needs to estimate
outstanding reported claims and IBNR requirements as at 31 December 2012 for the
purposes of the company’s year-end accounts. The company accounts for its business
on a one-year accounting basis.

In addition to the company’s own detailed policy and claims information, it has been
decided to obtain benchmark claims development and loss ratio information from
publicly available sources. The information available is paid, incurred and ultimate
loss ratios by year of account submitted by 10 different insurance companies. The
specific 10 companies were chosen because they provide a professional indemnity
classification within their returns.

Having collected this information it has been realised that it may not be appropriate to
include data relating to all of the 10 companies in the benchmark review.

(i) Explain the distinguishing aspects of each professional indemnity account that
should be investigated when selecting the most appropriate loss ratio
benchmarks for reserving purposes. [12]

The following are the selections made, weighted by volumes of business:

2009 Returns 2010 Returns 2011 Returns


PLR ILR ULR PLR ILR ULR PLR ILR ULR

2008 20% 89% 120% 40% 108% 125% 83% 120% 122%
2009 6% 34% 89% 22% 65% 85% 41% 75% 87%
2010 7% 32% 69% 23% 42% 65%
2011 3% 23% 59%

PLR = paid loss ratio


ILR = incurred loss ratio
ULR = ultimate loss ratio
(Note that the ILR does not include IBNR or IBNER claims)

Rate changes over the past four years have been provided by the underwriter and are
as shown in the table below.

Underwriting Years Premium Rate Change:

2008–2009 + 50%
2009–2010 + 25%
2010–2011 +10%
2011–2012 −5%

ST7 S2013–4
(ii) Describe how the information provided in the tables above might be used to
estimate a benchmark ultimate loss ratio for the 2012 accident year, stating the
additional information that might be required from the underwriter. [8]

(iii) Estimate a benchmark 2012 ultimate loss ratio using the information provided
and stating any other assumptions used. [4]

(iv) (a) Estimate average paid and incurred claims cumulative development
percentages for a typical accident year at the end of the first, second,
third and fourth development years.

(b) Comment briefly on the reliability of your results.


[7]
[Total 31]

END OF PAPER

ST7 S2013–5
INSTITUTE AND FACULTY OF ACTUARIES

EXAMINERS’ REPORT
September 2013 examinations

Subject ST7 – General Insurance:


Reserving and Capital Modelling
Specialist Technical

Introduction

The Examiners’ Report is written by the Principal Examiner with the aim of helping
candidates, both those who are sitting the examination for the first time and using past papers
as a revision aid and also those who have previously failed the subject.

The Examiners are charged by Council with examining the published syllabus. The
Examiners have access to the Core Reading, which is designed to interpret the syllabus, and
will generally base questions around it but are not required to examine the content of Core
Reading specifically or exclusively.

For numerical questions the Examiners’ preferred approach to the solution is reproduced in
this report; other valid approaches are given appropriate credit. For essay-style questions,
particularly the open-ended questions in the later subjects, the report may contain more points
than the Examiners will expect from a solution that scores full marks.

The report is written based on the legislative and regulatory context pertaining to the date that
the examination was set. Candidates should take into account the possibility that
circumstances may have changed if using these reports for revision.

D C Bowie
Chairman of the Board of Examiners

January 2014

 Institute and Faculty of Actuaries


Subject ST7 (General Insurance: Reserving and Practical Modelling Specialist Technical)
– Examiners’ Report, September 2013

General comments on Subject ST7

Candidates who are well prepared generally appear to perform reasonably on ST7, with the
more challenging questions tending to occur on SA3. Candidates should consider the
following advice however (if they are not already):

 Lists are hugely valuable for breadth of point generation but candidates should always
exercise judgement when applying them.

 Calculation questions will come up on a regular basis with ST7, as candidates can clearly
observe from examination of historical papers. Candidates should always be prepared for
such staples as balance sheet preparation, triangle manipulations & projections and
reinsurance layer calculations (along with being able to carry out any necessary
adjustments including inflation, exposure and time period issues).

 Capital questions should be expected on every paper and represent a sufficient proportion
of the course content that candidates should not expect to be able to pass on their
reserving knowledge alone. Those who do not encounter capital work in their
professional lives should be particularly careful to ensure that they take time to
familiarise themselves with this element of the course.

 Candidates should aim to be able to give near exact glossary definitions as incoherent or
vague descriptions will be marked harshly. If candidates struggle to remember definitions
verbatim they should take the time to properly analyse the glossary definition to ensure
they have fully absorbed all the nuances of the definition.

 It is important to always read the question properly.

Comments on the September 2013 paper

See comments under individual questions.


Subject ST7 (General Insurance: Reserving and Practical Modelling Specialist Technical)
– Examiners’ Report, September 2013

1 Main difference is either the entity that will make a claim under the policy or whether
coverage is for illegal versus inappropriate acts.

Fidelity guarantee is insurance for a company against dishonest action of employees


..such as fraud/theft /embezzlement that causes loss to the company.

While directors’ and officers’ liability provides coverage of directors and officers for
claims from third parties
...for losses caused by wrongful acts performed in their capacity as directors or
officers.

Candidates should have got full marks for this bookwork question. A good number of
candidates did so.

2 The prime objective for asset choice is to maximise investment return subject to
meeting all contractual obligations and recognising the uncertainties involved.
The implication for asset choice is that the assets should match those of the liabilities
in relation to term, amount, nature and currency.
Value for money/expected return: the price is dependent on whether attractive to
investors
…e.g. to pension schemes which may have pushed the price up to an unattractive
level

Product liability covers faulty design, faulty manufacture, faulty packaging and
incorrect instructions.
… for property damage and bodily injury
… usually on a claims incurred basis (rather than relating to year of sales).

There are some long reporting and settlement delays.


.. e.g. where time is taken to attribute physical conditions to the correct cause and
where claims relate to rogue drugs causing settlements to continue for some years or
any other reasonable example.

The suggested asset is long-term


However, the mean term of the asset is likely to be considerably longer than the mean
term of the liabilities.

The suggested asset purchase is appropriate by reference to the real nature of some of
the liabilities caused by the delays.

Bodily injury claims inflation might well be higher than CPI

The liabilities are volatile so difficult to match to assets in terms of cash flow

Expected return on this asset is likely to be lower than that which can be earned on
equities
If there are sufficient free reserves the asset may be suitable for part of these as less
volatile than equities.

Page 3
Subject ST7 (General Insurance: Reserving and Practical Modelling Specialist Technical)
– Examiners’ Report, September 2013

And depends on the risk appetite of the company

Does this asset fit in with the portfolio currently held?

What other classes of business are written and what percentage is this of the total
business (materiality)?

What currency are claims expected to be paid in?

Marketability in respect of accumulation of claims

Regulatory needs/ capital aspects

The credit rating is likely to be good as it is government debt


… but this does need to be considered particularly as the question doesn’t say what
government this refers to

Regional mix of business as local RPI may differ from territories where business is
written or where products are sold or manufactured
…even if only locally written, cases may be dragged to other jurisdictions

There may be diversification benefits as could be negative correlation between this


and other assets

Tax implications

On balance, argue whether or not worth buying

Generally well answered although a large number of candidates thought that the 1% nominal
coupon was the yield without realising that the actual yield depends on the price. Most just gave
advantages and disadvantages without actually specifying whether the stock should be purchased as
requested in the question.

3 (i) Dividend yields


Future interest rates/yield curves
Levels of major share indices
Future inflation rates
..could be split into RPI/CPI/wage inflation
Returns on property
Bond yields
Exchange rates
Credit spreads
Information on derivative contracts e.g. option price volatilities
Projected future default rates
Projected unemployment rates by country
GDP

Most candidates did not list sufficient variables to gain full marks.

Page 4
Subject ST7 (General Insurance: Reserving and Practical Modelling Specialist Technical)
– Examiners’ Report, September 2013

(ii) Fall in equity/property values due to recession..


..could also lead to increase in fraudulent claims in some classes.
..and higher losses from creditor/MIG policies.

Higher inflation rates will lead to changes in real bond yields..


..and higher claim costs in certain classes.
..together with increase in expenses/running costs for the business.

Large catastrophe event leading to high level of claims


..and potential problems recovering from reinsurers.
..and reduction in value of assets (or other example)

Significant changes in tax system in a particular territory


..could increase cost of settling claims and decrease post tax returns

The company may have investments in companies whose collapse could lead
to insurance as well as investment losses
..for example if creditor policies are offered to employees of a particular large
company.

Mis-valuation of assets could also mean mis-valuation of the reserves


...perhaps due to systematic control and governance problems in parts of the
company.

Other reasonable suggestions.

A large number of candidates did not read the question and rather than suggesting
interdependencies between assets and liabilities answered how to match assets and liabilities
or explained what dependencies they may have on external factors such as interest rates.

(iii) A central control system for all assumptions used in the model should be
maintained centrally
..as accuracy and consistency of assumptions is vital
..across different territories
.. within the company
.. and over time

Model change policy should be put in place


..with tight governance process for approval of change in assumptions
.. and acceptance control

Stakeholders in key assumptions should be part of the parameterisation and


sign-off process where possible
.. subject to appropriate sign-off authorities

All assumptions should ideally be validated


.. using historic data or benchmarks
..and documented fully including rationale for selection.
Data used should be up to date and appropriate

Page 5
Subject ST7 (General Insurance: Reserving and Practical Modelling Specialist Technical)
– Examiners’ Report, September 2013

All assumptions should be reviewed on a regular basis (e.g. annually)


..with independent/external validation being used as appropriate.

Professional guidance and regulations should be adhered to


Model input and output should be monitored

Process for communicating uncertainty to stakeholders.

A surprising number of poor or non-attempts given that some application of general


principles and common sense would give many of the points around consistency,
documentation and verification of assumptions. Many did not mention any governance
processes or sign-off requirements.

4 (i) Intermediary networks likely to be used widely to reach customers buying


houses.
..examples could be banks/building societies/conveyancers/homebuilders/
mortgage brokers.

Likely distributed as an affinity product with branding of bank/building


society
Possible to use independent brokers but may be difficult to reach the small
target market

Will not be financially viable for the company to set up a network of tied
brokers..
..but could sell direct through call centres if staff sufficiently skilled.

Internet/ door to door/ off the page unlikely to be appropriate


.. given possible high level of premium and length of commitment from
policyholder.

At point of sale an up-to-date valuation will be available but not necessarily


later.

Many candidates gave all possible distribution channels for a general insurance product
without demonstrating that they understood which would be appropriate for the specific
product. Very few pointed out that at point of sale there would be an up-to-date valuation but
not necessarily later.

(ii) Claims will be highly correlated with economic downturn


..so claims experience has potential to be very high if severe/prolonged
economic crash.
..and may be recessionary links to other risk factors

Doesn’t appear to be cap on potential downside...


..so potential losses up to value of each property.

Page 6
Subject ST7 (General Insurance: Reserving and Practical Modelling Specialist Technical)
– Examiners’ Report, September 2013

Single premium difficult to calculate given long term of contract


..especially uncertain as to amount of investment income on single premium.

New product so likely to be little historic data available on performance of


product.

Potential for adverse selection against the insurer as insureds could take
advantage of market developments to trade up
.. and could be moral hazard as insureds allow property to deteriorate if not
fully allowed for in valuation

Valuations to determine the extent of loss are likely to be subject to challenge


.. and may be biased/cyclical

To make product economic likely to require detailed terms & conditions


.. as otherwise would have high risk of insured taking advantage e.g. by
increasing mortgage amount or term or having mortgage holiday
.. this increasing the risk of mis-selling the product.
..particularly if it is being sold as an add on with other financial services e.g.
mortgage
..therefore risk of regulatory intervention if unhappy with product.

Premium likely to be very large as proportion of house sale


..so could be unpopular/unaffordable to customer meaning low take up
..then risk of company not being able to pay marketing/overhead costs.

Long term and volatile nature of product may require significant capital to
support

Some generic points although not specific, as requested, may be considered to


be more relevant for such a product: e.g. reinsurance may not be available or
expensive for such a product, new business strain, accumulations in particular
areas etc.

Many candidates gave generic risks rather than the specific risks requested in the question.

5 The first is model uncertainty.

This arises because actuarial models are often a simplification of a very complex (and
unknown) underlying system.
By using a simplified model to project the true underlying system, we are introducing
an unknown bias into the model.

Any reasonable example e.g. we may use standard probability distributions to model
claims size
whereas in reality the claims-size distribution will be unique and will depend on the
characteristics of the underlying book of business.

Page 7
Subject ST7 (General Insurance: Reserving and Practical Modelling Specialist Technical)
– Examiners’ Report, September 2013

The second is parameter uncertainty.

Parameter uncertainty refers to the uncertainty in determining the parameters for an


actuarial model.
This usually results from the statistical variability present in the historical data used to
estimate the parameters.

Any reasonable example e.g. an absence of large losses in historical data can lead to
an error in the estimation of the “average” claim development pattern.
Past data will never comprise all possible outcomes.

The third is process uncertainty.

If a process is assumed to be inherently stochastic, the future outcome will be


uncertain because of the randomness of the process and the fact of course that many
of these events have yet to occur.
This uncertainty is present even if model selection is perfect and the parameters are
known with certainty.

Any reasonable example e.g. a significant change in legislation with retrospective


effect may mean that even if the model exactly reflects the underlying process as it
existed in the past and it has been perfectly parameterised
the actuarial estimates do not reflect actual future events.

For a bookwork question this was poorly answered. Many candidates identified the three
main risks but were then generally unable to give much in the way of additional detail.
Despite specifically asking for examples many candidates failed to give these or gave
inappropriate ones. Also the definition of model risk was often wrong.

6 It should be noted that the selection of reserves for the accounts of an insurance
company are wholly the responsibility of the company’s board.

The changes make the selection of reserves more difficult. There are two extremes
that are possible: to ignore the changes entirely
and to assume that they will be fully effective

The actuary should ensure that in his report to the board the choice is set out clearly
so that the board may make an informed decision
..to select one of the two extremes
..or somewhere in between them
..and may give a range of possible outcomes
..and should stress that the reforms have not yet taken place
..and that will not affect claims already paid

It is not appropriate simply to be cautious: if it is appropriate to select a cautious


estimate then it is the board’s prerogative to do so, not the actuary’s, and the board
should be able to make such a choice in the informed knowledge that that is what they
are doing.

Page 8
Subject ST7 (General Insurance: Reserving and Practical Modelling Specialist Technical)
– Examiners’ Report, September 2013

There could be additional claims handling expenses which should be incorporated

The report should indicate how much confidence the actuary has in the changes being
successful,
..and the uncertainty should be clearly documented
..and the board will also be able to take into account reports about the reforms from
other functions in the company.

The actuary may indicate his preferred selection,


but this is one situation in which it may be appropriate for him not to do so.

Projecting ultimate claims on the basis that the reforms have no effect will present no
particular difficulties: the work should be done as if no changes had been made
..and will need to be done first, before considering any possible adjustments based on
the new procedures.

The normal difficulties of projecting ultimate claims will remain, but these would
have arisen in any case.

In projecting ultimate claims on the basis that the reforms are fully effective the
actuary will need to consider whatever evidence is available for their effect.

Work has clearly been carried out on the reforms, and this should have included an
evaluation of their effect on overall claims costs; it is possible that the actuary was
involved in this work.

If so, he may give it full credibility.

If, on the other hand, the quantitative work was done by, for example, claims
managers, then the actuary may examine it critically for methodological errors or
excessive optimism.
The actuary should also consult with other stakeholders
..and should challenge any assumptions that do not seem credible

It is unlikely that the unpaid claims will get the full benefit of any reforms.

Unpaid claims consist disproportionately of those that take a long time to pay.

These are claims that are not easy to settle, possibly requiring litigation, and may be
ones that were being more rigorously treated in the first place.

It may also be harder to start to deal rigorously with claims that had been opened and
initially dealt with in a more sloppy fashion than it is to deal with new claims
rigorously.

For example, evidence gathering may be difficult if not done early in the process.

Page 9
Subject ST7 (General Insurance: Reserving and Practical Modelling Specialist Technical)
– Examiners’ Report, September 2013

If the likely effect of the reforms has not been quantitatively assessed then the actuary
has little to go on to assess their likely effect. Caution is appropriate in these
circumstances.

It would be wrong to ignore entirely the possible effects of these reforms: they are
really happening and are evidently a significant development.

Some analysis of the projected savings, for example looking at the projected saving
by type of claim and length of time to settlement may be helpful, but ultimately this
exercise must depend heavily on the actuary’s judgement,
which must be clearly explained to the appropriate decision makers.

In effect the projection of results on the basis of no change and full success is a form
of scenario testing.

However, the actuary should not use this as a reason for testing wholly implausible
scenarios,
for example that the reforms have their full hoped-for level; or success on a book of
claims that is already partly processed.

Should consider any regulatory requirements/ tax guidance.


Must comply with professional standards
Speeding up payments will not necessarily reduce amounts but will affect
discounting.

Should ensure monitoring of the changes when they take place

Although the question does not ask how the actuary would adjust methodology to
allow for the new procedures marks are given for any reasonable generic
methodology points.

This was very poorly answered. Almost all candidates answered “how to adjust
methodology” rather than “what issues the actuary should consider” and “extent to which
he should allow for them” as asked for by the question. Even then there was some confusion
with many candidates seemingly answering the question given in April 2013 rather than
addressing the situation actually asked. Very few grasped the fact that the board set the
reserves.

7 (i) Ability to write business in many countries (without seeking separate


regulatory approval).

Lloyd’s chain of security including central fund can improve financial


standing
..which may be attractive to policyholders increasing business..
..as will the Lloyd’s of London brand and reputation.
There are stricter capital requirements
Lloyd’s has a high credit rating

Page 10
Subject ST7 (General Insurance: Reserving and Practical Modelling Specialist Technical)
– Examiners’ Report, September 2013

Access to large subscription market...


..writing a large range of classes of business
..unusual risks
..very large risks

Following on line-slips allows companies to build up experience and data over


time

Increased administrative requirements to comply with Lloyd’s regulation..


e.g. require to have an annual SAO which may be costly

A levy is payable for operating in Lloyd’s

Lloyd’s can impose restrictions on types of business written


..and approves/amends individual syndicate’s business plans.

Capital setting process is often perceived as unfair


Three year accounting basis/ profit not released until end of third year.

A bookwork question that was generally well answered although few scored the full marks
which were available.

(ii) The amounts of capital required are significant


..which indicates that the syndicate is probably large...
..or writes business that is particularly risky

In particular reserving risk is very high..


..possibly due to writing a significant amount of long tailed business
..and/or reflecting that the syndicate has been in operation for some time

Underwriting risk is also material


...so the syndicate intends to continue to write a significant amount of business
..perhaps increasing proportion written in capital intensive areas such as
catastrophe reinsurance
..though given size of reserving risk could have been larger in the past and
shrunk

The aggregate insurance risk of 550 is not possible if other figures are correct
(and assuming that insurance risk comprises only reserving and underwriting
risk)
..even if risks were fully dependent would give maximum capital requirement
of 525.
..indicating that the correlations in the model are incorrect or some other error.
Or that there are other contributing components not shown in the table (e.g.
catastrophe risk)

Market risk is relatively low...


..as would typically be the case for a Lloyd’s syndicate.
..indicating investments are probably low risk such as gilts rather than equities

Page 11
Subject ST7 (General Insurance: Reserving and Practical Modelling Specialist Technical)
– Examiners’ Report, September 2013

Counterparty default risk is unusually high


..suggesting that significant reinsurance
..from lower grade reinsurers may be utilised
..and/or significant funds remain with third parties (brokers/loss funds at
MGAs)
..and/or investment counterparties may be weak.
There is some overlap between market risk and counterparty risk so relativities
may or may not be odd.

Operational risk appears to be low relative to other risks


..possibly due to inadequacies in modelling for this risk

As expected, liquidity risk is fairly low as secure investments...


...and future premium income from new business.

The total syndicate capital requirement seems quite low compared to its
constituent parts
..again suggesting issues with correlation assumptions or application
..or alternatively that the total figure is discounted whilst components are not.

Calculation of fully diversified syndicate capital requirement (£433m) (square


root of sum of squares of each assumed independent risk component)
...compared to assuming complete dependence (£685m)
Note: other assumptions could be used and hence different answers obtained
and should be given marks if reasonable explanation (e.g. using the £550
aggregate risk figure given if assumption that includes other components).

No group risk indicates that syndicate is stand-alone (unless omitted, given


other problems with the figures).

It would be useful to know on what basis the figures have been calculated
..and the mix of business written

Many candidates scored quite well. Surprisingly few however commented on the absolute size
of the capital figure indicating a large syndicate, questioned on what basis the figures had
been prepared, or, having commented on reserving risk, made any comments on underwriting
risk. Also, many candidates failed to spot the obvious errors/inconsistencies in the table
despite the question asking to consider “reasonableness”, with some coming up with long
winded explanations as to why the figures made sense.

(iii) Amend business plan of what is to be written over the next year
..either by reducing volumes written
..or moving to less capital intensive or uncorrelated lines
May be difficult to achieve much relief from this as reserves are dominant
Reduction may be problematic in any case as fixed costs may rely on a level
of current year activity

Alternatively could increase volumes of business written to give additional


profits assuming reserving risk is the dominant factor

Page 12
Subject ST7 (General Insurance: Reserving and Practical Modelling Specialist Technical)
– Examiners’ Report, September 2013

Seek additional sources of capital from elsewhere


...such as from the names
…taking on a third party capital provider or otherwise raising finance

Purchase additional reinsurance from highly rated reinsurers


..in particularly entering into a quota share arrangement can provide capital
relief
..or require reinsurers to post collateral

Attempt to reduce reserving risk by undertaking commutations/LPT/novation


on back-year liabilities
..though this option is likely to be costly and time consuming

Engage third party actuary to carry out a review of the capital model.
..to provide independent validation that the estimated capital is reasonable.

Hold discussions with the regulator to identify any areas of difference arising
from particular risks..
..and provide additional information if there are syndicate specific issues that
are not being considered.

Improve market and liquidity risk e.g. by increasing matching

Should combine different mitigating actions


..but should also consider materiality (it is not stated how much more capital is
required)

Most candidates got the more generic points such as reducing business written, more
reinsurance etc. Some candidates talked about one or two possibilities in great detail (e.g.
changing the reinsurance). Answers such as using exotic reinsurance or investment products
would not be appropriate as these would increase risk. Equally the regulator is unlikely to
find a response involving the wholesale revaluation of reserves or assets such as allowing for
discounting acceptable. Not many identified challenging Lloyd’s review or getting an
independent review.

8 (i) Mix of business by profession

The ultimate loss ratios (ULR’s) may differ between, for example, solicitors,
architects, brokers, actuaries, accountants, IFAs.

This may be due to different levels of competition in these different sectors.

Due to different levels of risk appetite


e.g. architects’ and solicitors’ professional indemnity experience have been
poor in recent years (or other relevant example)

Some insurers may concentrate on only one profession

Page 13
Subject ST7 (General Insurance: Reserving and Practical Modelling Specialist Technical)
– Examiners’ Report, September 2013

Others may exclude certain professions e.g. no big-4 accountancy firms

Some professions may incept at certain times of the year e.g. solicitors
business incepting in September/October
which is particularly relevant to consider for a company that has only been
writing since 1 July.

Size (turnover or number of partners) of assureds

Large practices may have very different claims experience to smaller


practices.

Higher fees and larger potential losses.

Different risk management standards.

Different clients and therefore different services provided (some more risky).

Territory in which business written

London market business may be different in nature to regional e.g. due to size
of risks.
London market business will include world-wide risks.

US professional indemnity business in particular may perform differently to


local

Size and type of lines written

Some insurers may write mainly primary layers whereas others write excess
layers.

Different terms and conditions


..e.g. limits and deductibles may differ.

Claims made versus claims occurring policies: business written on a claims


made basis may be different in nature to that written on a claim occurring
basis.
Would expect a slower claims development pattern on a claims occurring
policy than on a claims made policy.
Although professional indemnity is generally written on a claims made basis

Accident or underwriting year basis


Premiums net or gross of commission
..and net or gross of reinsurance
If net of reinsurance need to consider differences between reinsurance
programmes

Page 14
Subject ST7 (General Insurance: Reserving and Practical Modelling Specialist Technical)
– Examiners’ Report, September 2013

Different underwriting philosophies

Some insurers trying to build market share may be happier to write at higher
loss ratio.

Differing quality of underwriting done by competitors.

Different reserving philosophy: Some insurers may reserve strongly, others


less so.
May appear in returns as by underwriting year or accident year.

Underwriting year claims development expected to be slower than accident


year development.
Size of account impacts credibility for benchmarking purposes.

Smaller accounts may be more volatile in terms of claims experience.

Large claims may distort the benchmarks

Reputation, age and experience of the company


Distribution channels
Different claims management processes

However, there is a need to be pragmatic as making too many restrictions will


mean that there are very few or no companies left to benchmark against

Many candidates managed to identify quite a range of issues that should be taken into
account. Not many commented on the original insureds’ risk profile or gave examples of
different professions. Hardly anyone commented on the credibility of the data given the
number of potential restrictions.

(ii) Professional indemnity is reasonably long-tailed so place less reliance on


ULR’s on immature years.
.. although in the figures given the 1st year estimates appear to be reasonably
reliably forecast
.. and as they are the most recent forecasts are not to be taken lightly

At the end of the first development year, the incurred claims seems only to be
about 40% of ultimate claims.

So suggest not relying on 2011 booked ultimate loss ratio.

Historically, it can be seen that the ULR for a particular accident year can drift
over time.

It may be that companies book a pessimistic ULR initially to avoid poor run-
off.

Page 15
Subject ST7 (General Insurance: Reserving and Practical Modelling Specialist Technical)
– Examiners’ Report, September 2013

2008 may be too old to be representative


.. .terms and conditions are likely to have changed alongside the premium rate
changes
and there may have been a shift in the mix of business with new capacity
entering the market.

Establish from underwriter whether there are any large losses/events that
might have distorted any of the figures in the market information.

Although these may be heavily skewed based on changing environmental/


legislative/ economic factors depending on the business written

May use 2009 and/or 2010 accident years as starting points (or other
suggestion, if reasonable).

Roll forward for premium rate changes


Definitely need to adjust for rate changes as they haven’t been flat.
And adjust for inflation, if appropriate

Need to understand from underwriter more about what the premium rate
change information represents
... .before or after allowing for claims inflation?
... .and what would a typical claims inflation assumption be?
.. ..allowance for changes in terms and conditions e.g. reduced coverage?

Should consider whether the premium rate changes are appropriate


May estimate the earned premium rate change between year x and x + 1 as the
average of the underwriting year rate changes between x 1 and x + 1.

Although would discuss with underwriter how business incepts and earns
throughout the year.
..and obtain underwriters’ views on the loss ratios

Should also enquire as to when information updated with 2012 data may
become available

Most candidates suggested rolling forward for rate changes but few considered other issues
such as which years to use with some not even mentioning inflation. Few commented on the
actual figures in the table, e.g. on the first year estimates being quite good. A few candidates
suggested using a Bornhuetter-Ferguson method for obtaining the benchmark which shows a
complete lack of understanding.

Page 16
Subject ST7 (General Insurance: Reserving and Practical Modelling Specialist Technical)
– Examiners’ Report, September 2013

(iii) The assumption from how the question is written should be that the returns are
based on account year data and therefore the rate changes need to be adjusted
to an underwriting year basis before rolling forward to 2012 terms. Also, if
2008 is included then an assumption needs to be made for the earned rate
change for that year:

Accident Year .. 2008 2009 2010 2011


Starting ULR (1) 122% 87% 65% 59%
Earned rate change to next
(2) 50% 37.5% 17.5% 2.50%
year
Rate change factor to 2012 (3) 248% 166% 120% 103%
Inflation to 2012 (4) 136% 126% 117% 108%
ULR in 2012 terms =(1)(4)/(3) 67% 66% 63% 62%

Sensible claims inflation assumption e.g. 5% to 10%, or zero if it is


specifically stated that the rate changes allow for inflation and that the level of
inflation used is reasonable

Select the average of the ULRs => 64% based on years 2009 to 2011 or 65%
based on all 4 years (or other sensible selection).

Despite the assumption from how the question is written candidates may have
assumed that the returns are based on underwriting year data and therefore
the rate changes do not need to be adjusted, but do need to state this and use
half-year less for the inflation factors to get to account year 2012:

Underwriting Year 2008 2009 2010 2011


Starting ULR (1) 122% 87% 65% 59%
Earned rate change to next
(2) 50% 25% 10% -5%
year
Rate change factor to 2012 (3) 196% 131% 105% 95%
Inflation to 2012 (4) 131% 121% 112% 104%
ULR in 2012 terms =(1)(4)/(3) 82% 81% 70% 65%

Specific assumption made that figures given are on an underwriting year basis
Sensible claims inflation assumption e.g. 5% to 10%, or zero if it is
specifically stated that the rate changes allow for inflation and that the level of
inflation used is reasonable

Select the average of the ULRs => 72% based on years 2009 to 2011 or 74%
based on all 4 years (or other sensible selection).

This was very poorly answered. Despite often stating in the answer to (ii) that allowance for
inflation should be made very few actually did so. The assumption could have been made that
the rate changes allowed for inflation but very few made such an assumption some even
assuming 0% inflation. Often candidates tried to project the incurred claims data to ultimate
although ultimates have already been given. Some projected the triangle of ultimates which
might have been appropriate if there was a trend but there was no evidence of this. Despite the

Page 17
Subject ST7 (General Insurance: Reserving and Practical Modelling Specialist Technical)
– Examiners’ Report, September 2013

question asking for an accident year figure while rate changes are shown on an underwriting
year basis hardly anyone made any allowance for this.

(iv) Divide PLR and ILR by ULR to get sample percentage development:

PLR/ULR

Ac 1st 2nd 3rd 4th


Yr\Dyr

2008 16.7% 32.0% 68.0%


2009 6.7% 25.9% 47.1%
2010 10.1% 35.4%
2011 5.1%

ILR/ULR

Ac 1st 2nd 3rd 4th


Yr\Dyr

2008 74.2% 86.4% 98.4%


2009 38.2% 76.5% 86.2%
2010 46.4% 64.6%
2011 39.0%

Note: an alternative approach to the above calculations is to divide the PLR or


ILR by the latest estimate of the ULR in each case, rather than the ULR
estimated at previous year-ends. This approach assumes that the earned
premium in the denominator is consistent over time (i.e. no late bookings or
mis-statements of premium). This alternative approach should get credit if that
assumption is stated.

PLR/Latest ULR

Ac 1st 2nd 3rd 4th


Yr\Dyr

2008 16.4% 32.8% 68.0%


2009 6.9% 25.3% 47.1%
2010 10.8% 35.4%
2011 5.1%

Page 18
Subject ST7 (General Insurance: Reserving and Practical Modelling Specialist Technical)
– Examiners’ Report, September 2013

ILR/Latest ULR

Ac 1st 2nd 3rd 4th


Yr\Dyr

2008 73.0% 88.5% 98.4%


2009 39.1% 74.7% 86.2%
2010 49.2% 64.6%
2011 39.0%

Identification of correct development year against each data point.

Average development %’s across accident years are as follows:

.. ULR .. Latest .. ..
basis ULR
basis
Dev Yr Paid Incurred Paid Incurred ..

1 7.3% 41.2% 7.6% 42.4%


2 26.0% 71.8% 25.7% 70.8%
3 39.6% 86.3% 40.0% 87.4%
4 68.0% 98.4% 68.0% 98.4%

Dev year 4 position is based on only one sample point (relating to 2011) so
this is likely to be less reliable.

For incurred claims, there seems to be a reasonable amount of consistency


between sample points at the same development period

For paid claims, there appears to be some evidence of claims speeding up


..for second dev year, % developed goes up from 17% on 2008 accident year
to 26% on 2009 and 35% on 2010.
..for third dev year, % developed goes up from 32% on 2008 accident year to
47% on 2009.
So using averages won’t reflect any such trends.
So may prefer to use incurred patterns rather than paid patterns (especially as
incurred is more mature).

Any other reasonable observations.

Again very poorly answered. Many candidates failed to realise that the paid/incurred loss
ratios should be divided by the ULRs. Many candidates did not attempt the question or made
a complete mess of the calculations often giving answers which were obviously wrong
without comment e.g. cumulative development percentages at 4th year of 100% for both paid
and incurred claims. Comments on reliability often tended to make little reference to the
figures.
END OF EXAMINERS’ REPORT

Page 19
INSTITUTE AND FACULTY OF ACTUARIES

EXAMINATION

2 May 2014 (pm)

Subject ST7 – General Insurance:


Reserving and Capital Modelling
Special Technical

Time allowed: Three hours

INSTRUCTIONS TO THE CANDIDATE

1. Enter all the candidate and examination details as requested on the front of your answer
booklet.

2. You have 15 minutes at the start of the examination in which to read the questions.
You are strongly encouraged to use this time for reading only, but notes may be made.
You then have three hours to complete the paper.

3. You must not start writing your answers in the booklet until instructed to do so by the
supervisor.

4. Mark allocations are shown in brackets.

5. Attempt all five questions, beginning your answer to each question on a new page.

6. Candidates should show calculations where this is appropriate.

AT THE END OF THE EXAMINATION

Hand in BOTH your answer booklet, with any additional sheets firmly attached, and this
question paper.

In addition to this paper you should have available the 2002 edition of the Formulae
and Tables and your own electronic calculator from the approved list.

ST7 A2014 Institute and Faculty of Actuaries


1 A general insurance company determines its capital requirements using a stochastic
model that it develops itself.

Discuss the two approaches that it might take for the treatment of its outwards
reinsurance programme within this model, including:

• the advantages and disadvantages of each such approach.


• the implications for the development of other parts of the model.
[12]

2 In a certain country, general insurance companies are not allowed to merge unless
they have commissioned a report for the country’s insurance regulator from an
independent actuary that concludes that the merger will not significantly disadvantage
the policyholders of either company. All insurance companies in the country
concerned are proprietary companies owned by shareholders.

An actuary is in the process of investigating a merger in order to decide whether he is


able to provide such a report.

Discuss the matters that the actuary should consider and the investigations that should
be undertaken. [12]

3 A consortium of billionaires has decided to form a company to mine asteroids.


Investigations have shown that the Earth’s resources of precious minerals are being
depleted and that the asteroids contain immense amounts of such minerals. The
proposal is to use space-going rocket-ships to make very large profits by mining these
resources and bringing them back to Earth.

The consortium has employed experts to plan the project and has concluded that the
return on capital would be higher than any alternative projects, subject to any
insurance considerations.

Discuss the risks for the company and the types of general insurance cover that it
might require. [22]

ST7 A2014–2
4 A large multi-national insurance group is intending to create a subsidiary insurance
company, Stop Loss Re, to write solely whole account stop loss reinsurance business
of world-wide general insurance and reinsurance companies.

It has been a widely held belief that stop loss reinsurance is either unobtainable or
over-priced. The rationale for Stop Loss Re is that by writing just stop loss it will be
the expert in the field and therefore will be able to make stop loss insurable at a
reasonable price while making a worthwhile return on the capital invested.

(i) (a) List the risks that should be considered for assessing the capital
requirements of an insurance company.

(b) Highlight which of these risks are of particular relevance to Stop Loss
Re. [15]

(ii) Describe the outwards reinsurance that Stop Loss Re should seek to purchase
and the difficulties involved. [6]
[Total 21]

ST7 A2014–3 PLEASE TURN OVER


5 (i) Define the three types of reserve a general insurance company may hold in
respect of an unexpired period of cover. [3]

A general insurance company writes personal lines home insurance business. For the
most recent four years it has written part of its book of home contents insurance with
a return of premium guarantee.

When a policyholder has purchased this contents insurance from the company for ten
consecutive years, at the end of the ten year period the company will repay 100% of
the premiums paid by the policyholder less any claims paid, if this calculation is
positive.

The premium for the first-year policy is at the company’s standard rates for contents
insurance plus a loading of 10%, this amount being the company’s estimate of the
loading required to cover the premium return guarantee. Renewal premiums and
sums insured are guaranteed to increase at the same rate as the local retail prices index
(RPI) as long as no claims have been incurred.

The following data for this class of business have been supplied by the company as at
31 December 2013:

Claims Paid (000s)

Uwyr\Dyr 1 2 3 4

2010 1,210 2,625 2,756 2,756


2011 1,817 3,938 4,121
2012 2,121 4,321
2013 2,075

Uwyr = underwriting year


Dyr = development year

Premiums (000s)

Uwyr Written Written Premium for


Premium: Policies Still Being
Total Renewed from Previous
Years

2010 2,675 1,375


2011 4,075 2,175
2012 4,335 3,127
2013 3,174 3,174

For example, for underwriting year 2010, of the 2,675 premium written in that year
1,375 of that amount is for policies that will have been renewed in each of 2011, 2012
and 2013. For underwriting year 2011, of the 4,075 premium written in that year
2,175 of that amount is for policies originally started in 2010 and 2011 that will have
been renewed in each of 2012 and 2013. Similarly for 2012 and 2013.

ST7 A2014–4
The claims paid for policies that have been renewed from previous years were all zero
at the 2013 renewal date.

RPI

Uwyr Increase
From-To in RPI

2010–11 5.72%
2011–12 6.17%
2012–13 1.50%

The lapse rate for the company’s standard contents insurance business has been 20%
per annum over recent years.
(ii) Calculate the outstanding claims reserve, unearned premium reserve and
additional unexpired risk reserve as at 31 December 2013 for this book of
business, stating the assumptions you make. [20]

(iii) Outline reasons why the assumptions made in part (ii) may not be correct and
any other problems with the estimation of reserves for this business. [10]
[Total 33]

END OF PAPER

ST7 A2014–5

You might also like